MCAT behavior

¡Supera tus tareas y exámenes ahora con Quizwiz!

Which of the following phrases spoken from a resident physician to his intern during patient rounds demonstrates alter-casting? A. "You, as her physician, should treat her with statins to decrease her cholesterol level." B. "As your resident, I'm here to help teach you the procedure properly." C. "Wow, your skills are becoming very sharp. Would you mind taking over two of my patients?" C. "I wasn't able to do your evaluation because I recently had an increase in my patient load."

(A) is a correct example of alter-casting, because the resident physician imposes an identity onto the intern by saying as her physician. (B) is an example of self-disclosure, because the resident physician emphasizes his own identity as your resident. (C) is an example of ingratiation, because the resident physician attempts to use flattery to get the intern to do extra work. (D) is an example of aligning actions, because the resident physician tries to use an excuse to make his behavior of not doing the evaluation acceptable. step by step Simplify the question This question addresses a scenario in which a resident physician speaks to his intern. We are presented with four phrases and have to identify which one of these phrases is an example of the impression management strategy alter-casting, so the question remains: Which of the following phrases from a resident physician best represents an example of alter-casting? What is known? Unknown? On Test Day, as motivated MCAT students, we should be familiar with all five impression management strategies. Here we have to find the best example of alter-casting. Alter-casting is an impression management strategy in which a person imposes an identity onto another person.You may realize that you have just been addressed as a motivated MCAT student. This means that Kaplan has assigned you an identity (motivated student). Other examples include: "You're such a great friend...;" "As a fantastic mother...;" "Wow, you are such a great parent...;" "You are such a wonderful professor...." The key in selecting the correct answer is going to be finding the specific phrase in which the resident physician imposes an identity onto his intern. What is your prediction? We are looking for a phrase which shows that the resident physician is assigning an identity to the intern: casting them in a role in which the resident wants the intern to see himself. In (A), the resident physician addresses the intern as if he already is a physician (You, as her physician...). That means an identity is imposed onto the intern which makes (A) the correct answer. In (B), the resident physician establishes his own identity (As your resident...), so this is not an example of alter-casting. In (C), the resident physician flatters the intern which represents an example of ingratiation, another impression management strategy. In (D), the resident physician justifies not having done an evaluation because he was busy.This best represents an excuse which is reminiscent of the impression management strategy of aligning actions. We can conclude that (A) is the best example of alter-casting.

Which of the following actions is NOT performed by the parasympathetic nervous system? A. Pupil dilation B. Decrease of heart rate C. Stimulation of bile secretion D. Bladder contraction.

Correct answer: A The parasympathetic nervous system opposes the sympathetic nervous system. It's actions can be summed up as "rest and digest". Thus it causes heart rate to slow (B) and bile to be released (C) . It is also involved in contraction of smooth muscles causing the bladder (D) as well as the iris to contract. Dilation (A) is the opposite of this contraction and thus is performed by the sympathetic nervous system, not the parasympathetic.

Which of the following pairs of experimental groups would best allow researchers to study the effects of social facilitation? A. Members of one group perform a simple task in front of three people; members of another group perform the same task in front of fifteen people. B. Members of one group perform a complex task alone; members of another group solicit help with the task from bystanders. C. Members of one group perform a simple task in front of fifteen people; members of another group perform a more complicated task in front of three people. D. Members of one group are asked to help a confederate perform a complex task; members of another group are asked to watch and evaluate the confederate's performance.

Correct answer: A *Social facilitation is the idea that people tend to perform better on simple tasks when in the presence of others.

Which of the following statements is correct? A. A church is an example of a Gemeinschaft because the members attend due to a shared interest in religion. B. A church is an example of a Gesellschaft because the members attend due to a shared interest in religion. C. South Africa is an example of a Gesellschaft because it has clear demarcations between its territory and that of its neighbors. D. South Africa is an example of a Gemeinschaft because the citizens of the country do not all share common beliefs and religion.

Correct answer: A A church has shared interests and beliefs among its members and thus qualifies as a Gemeinschaft, so (A) is correct. A church's shared interest in religion does not qualify as a Gesellschaft which requires shared objectives, thus (B) is incorrect. Organized territories and physical demarcations are not characteristics of a Gemeinschaft (C). Furthermore, a Gemeinschaft does share common beliefs, thus (D) is false regardless of whether South Africa qualifies as a Gemeinschaft. Only (A) is correct . Step by step Simplify the question We have to go straight to the answer choices to see the scope of this question which addresses the concepts of Gemeinschaft, a German term that translates as community, and Gesellschaft which translates as society. We are presented with four answer choices and have to identify which of these accurately describes a Gemeinschaft or Gesellschaft; therefore, the question becomes: Which of these answer choices is a correct example of Gemeinschaft or Gesellschaft? What is known? Unknown? The German sociologist Ferdinand Tönnies distinguished two major types of groups each with their own characteristics. Gemeinschaft (community)—Group unified by feelings of togetherness due to shared beliefs, ancestry, or geography; examples include close-knit neighborhoods, family, and friendship groups Gesellschaft (society)—Group unified by mutual self-interests in achieving goal; examples include companies and countries Now that we have refreshed our knowledge of Gemeinschaft and how it differs from Gesellschaft, we have to review the answer options in order to assess which one represents the best example. To remember the difference, we might think "ge-MINE-schaft" (it's mine, it belongs with me and I belong with it) and "ge-SELL-shaft" (we're a company or country, we sell things). What is your prediction? A church (A) is certainly a Gemeinschaft because the members have a shared belief system, and the church most likely provides people with a sense of belonging. For many people, churches provide a 'community feel' because most people know the other members of their church and socialize with them. Churches aren't typically focused by mutual self-interest on achieving a goal, so (B) is incorrect. As we have seen in Step 2, a country is an example of a Gesellschaft, so (D) is eliminated. (C) can be disregarded due to the associated rationale. Geographical borders are not a defining feature that make a country a Gesellschaft. To reiterate, (A) is the only correct answer. In depth The German sociologist Ferdinand Tönnies distinguished two major types of groups. His theory is known as Gemeinschaft und Gesellschaft which translates to community and society. Gemeinschaft (community) refers to groups unified by feelings of togetherness due to shared beliefs, ancestry, or geography. Families and neighborhoods are examples of Gemeinschaften. Gesellschaft (society) refers to groups that are formed because of mutual self-interests working together toward the same goal. Companies and countries are examples of Gesellschaften.

Which of the following is the best example of a manifest function of an institution? A. Afterschool childcare programs allow adults in single-parent households to maintain full-time employment. B. Elementary school students learn to navigate social circles through spending time with their peers. C. The legalization and taxation of marijuana decrease drug use among teenagers. D. Imprisoning individuals for minor offenses can cause them to lose their jobs and increase recidivism rates.

Correct answer: A A manifest function is an explicit, intended function of an institution whereas a latent function is an unintended function of an institution; thus, the correct answer is (A) which is an explicit, intended function of afterschool care. (B) and (C) are unintended but positive functions of their institutions; school is primarily for education, and drug legalization is primarily about decriminalization and a source of income. As such, these are latent functions. (D) also describes a latent function, but it is furthermore a dysfunction: an unintended and negative consequence of an institution. (A) is therefore correct. Step by step Simplify the question A manifest function results in an intended effect; thus, we can eliminate scenarios that describe a change resulting in an unexpected or unintentional result. When reading through each answer choice, consider what changes you might expect from the action and then compare what you come up with against the result stated in the answer choice. Another way to phrase this question would be: Which scenario best describes an intended result? What is known? Unknown? Background knowledge of the following definitions tells us that a manifest function has intended consequences while a latent function has unintended results. The unknown is whether the results presented in each answer choice would be expected or not given the intervention. Dysfunction—Harmful consequences of people's actions which undermine social system's equilibrium Function—Beneficial consequences of people's actions Latent function—Unexpected, unintended, or unrecognized positive consequences of manifest functions Manifest function—Deliberate actions that serve to help given system What is your prediction? The expected purpose of an afterschool care program would be that parents could complete a work day while knowing that their children are cared for, thus enabling them to hold full-time jobs. Because the resulting function described in the answer choice is the same as the expected purpose of the care programs, this is a manifest function; (A) is correct. The primary purpose of an elementary school experience would be for educational purposes; however, (B) describes the secondary social benefits—a latent function of elementary school education. The expected purpose of legalizing and taxation of marijuana sales would be for governments to gain better regulatory control over distribution of the substance, but (C) describes a concomitant decrease in marijuana use. This describes a latent function of the regulatory change. The expected function of imprisonment for minor offenses would be an attempt at reducing crime; (D) describes collateral job loss and increases in recidivism as a result of incarceration, otherwise known as a latent function. In summary, (A) is correct.

A student asks her professor for an extension on a research paper. Which of the following statements best represents the impression management strategy of alter-casting? A. "You're such a great professor, so I did not think it would be a big deal to ask." B. "My last professor was a great guy and gave me an extension whenever I needed it." C. "I need the extension because I was at the vet all day yesterday. My cat got sick." D. "I don't know if I've told anyone this before, but I have severe insomnia and it got in the way of my work."

Correct answer: A Alter-casting is imposing an identity onto another person which makes (A) correct. In (B), the student attempts to manipulate the professor by highlighting favorable characteristics of a former professor. (C) would be considered aligning actions (the use of excuses to account for questionable behavior). (D) is more reminiscent of self-disclosure. Only (A) is correct. Self-disclosure--->Giving information about oneself to establish an identity----> ex.Disclosing that you are a premedical student Managing appearances--->Using props, appearance, emotional expression, or associations with others to create a positive image--->ex. Wearing a white coat, keeping calm while dealing with a difficult patient, mentioning associations with important researchers during an interview Ingratiation--->Using flattery or conforming to expectations to win someone over ---> ex. Blindly agreeing to someone else's opinion, complimenting a friend before asking for a favor Aligning action--->Making questionable behavior acceptable through excuses---> ex. Justifications for missing deadlines, blaming a bad grade on too little sleep Alter-casting--->Imposing an identity onto another person---> ex. Any example in this course that says As a good MCAT student, you should... in which Kaplan is assigning you the role of good MCAT student

A heavy drinker begins to attend Alcoholics Anonymous meetings because he wants to learn how to control his behavior and improve his relationships with others. Which of the following best describes the desired transformation? A. He is making an attempt to align his authentic self with his ideal self. B. He is improving his tactical self so that he can avoid alcohol and how he responds when he sees it. C. He is aligning his ideal self to reality so that he experiences less cognitive dissonance. D. He is changing his authentic self to better match his tactical self.

Correct answer: A Authentic self— describes who the person actually is and includes both positive and negative attributes Tactical self— refers to who we market ourselves to be when we adhere to others' expectations of us Ideal self—refers to whom we would like to be under optimal circumstances Cognitive dissonance—the simultaneous presence of two opposing thoughts or opinions By now, we have reacquainted ourselves with the concepts that are articulated in the answer options. However, what we don't know yet is how these concepts apply to the scenario that is described in the question stem.

Which of the following is NOT considered a form of nonverbal communication? A. Sign language B. Eye contact C. Body language D. Touch.

Correct answer: A Choice (A) . Nonverbal communication refers to how individual communicate without the use to words. Since sign language uses gestures to represent words, it is classified as verbal communication and thus the correct choice.

Which of the following personality perspectives views personality as a product of an individual's unconscious urges and desires? A. Psychoanalytic perspective B. Humanistic perspective C. Social cognitive perspective D. Behaviorist perspective.

Correct answer: A Choice (A) . Personality theories that focus on how an individual's unconscious internal states influence their external behavior are said to follow a psychoanalytic perspective. Sigmund Freud's work with the id, ego, and superego illustrates a theory from a psychoanalytic perspective.

Many universities offer scholarships for individuals who are the first in their family to attend college. Since higher education is correlated with higher socioeconomic status, the scholarships likely result in which of the following? A. Upward mobility B. Meritocracy C. Strong ties D. Ascribed status.

Correct answer: A Choice (A) . Since higher education is associated with higher socioeconomic status, as well as social capital, the individual would be experiencing upward mobility. Upward mobility refers to any positive change in a person's status (economic or positional).Choice (B) . Meritocracy also refers to a change in social status, but refers specifically to a change based on the individual's unique talents and achievements.Choice (C) . Strong ties refers to an individual's peer group and kinship contacts.Choice (D) . Ascribed status refers to a position in a society based on characteristics beyond the individual's direct control; such as race, gender, ethnicity, and family background, and does not involve mobility.

A patient reports feeling uneasy in front of people and worries that it will adversely affect his position as a university lecturer. The patient also describes himself as a "wallflower" at parties. Which of the following diagnoses best addresses the patient's complaints? A. Anxiety Disorder B. Schizophrenia C. Mood disorder D. Personality disorder.

Correct answer: A Choice (A) . Social anxiety disorder is characterized by anxiety that manifests mostly in social situations, such as at a party or in front of a crowd. The disorder is more broadly characterized as an anxiety disorder.

Which of the following situations most clearly portrays conflict theory? A. Factory workers strike against factory owners because the workers are paid very little, but the owners need them. B. A society functions by splitting all of its resources evenly, regardless of the type of work a citizen performs. C. Two competing companies watch each other's actions and respond in order to increase sales. D. A company gives one candidate a job because the other applicant was considered too old for the job.

Correct answer: A Conflict theory—describes how power differentials are created and how these differentials contribute to the maintenance of social order Game theory—a model that explains social interaction and decision-making as a game, including strategies, incentives, and punishments Ageism— prejudice or discrimination on the basis of a person's age Conflict theory posits how power differentials are created and how these differentials contribute to the maintenance of social order. Further, power differentials can lead to the dominance of a particular group if it successfully outcompetes other groups for economic, political, and social resources. It is important to recognize that this is a macro theory that applies to larger societal bodies and is not about individuals, but about how larger parts of society struggle to get and maintain power in society. From this understanding we can eliminate (C) and (D) because the scope is too small. We can also eliminate (B) because of the lack of conflict. This leaves (A) as the correct answer, which is the only choice to demonstrate conflict between groups on a larger scale.

Which of the following describes the ability to see that some properties are unchanged after an object undergoes a physical transformation? A. Conservation B. Concrete operational C. Object permanence D. Symbolic thinking

Correct answer: A Conservation - A concept seen in quantitative analysis performed by a child; develops when a child is able to identify the difference between quantity by number and actual amount, especially when faced with identical quantities separated into varying pieces Concrete operational - This is one of Piaget's stages of development that lasts from about 7 to 11 years of age. In this stage, children can understand conservation and consider the perspectives of others. Object permanence - Knowledge that an object does not cease to exist even when the object cannot be seen; a milestone in cognitive development. Symbolic thinking - Refers to the ability to pretend, play make-believe, and have an imagination

Which of the following is NOT a type of explicit memory? A. Procedural memory B. Episodic memory C. Semantic memory D. Declarative memory.

Correct answer: A Explicit memory, also called declarative memory (D) , is the memory associated with conscious recall. This includes facts and concepts, as in semantic memory (C) , and events and experiences, as in episodic memory (B) . Procedural memory (A) is unconscious and necessary for skills involving muscle memory, such as riding a bicycle or throwing a ball.

In an experiment, participants are shown four random school children, two male and two female, and asked which one is most likely to have won the award for math and which one is most likely to have won the award for English. Eighty percent of the participants chose male students to win the math award, but only forty-five percent chose the male students to win the English award. Assuming these results are based on common stereotypes, which of the following phenomena is most likely responsible? A. Representativeness heuristic B. Confirmation bias C. Inductive reasoning D. Availability heuristic

Correct answer: A Heuristics are simplified principles used to make decisions; they are colloquially called rules of thumb. Representativeness heuristic - A shortcut in decision-making that relies on categorizing items on the basis of whether they fit the prototypical, stereotypical, or representative image of the category Confirmation bias - The idea that we lend more credence to information that fits into our already held beliefs Inductive reasoning - A form of cognition that utilizes generalizations to develop a theory Availability heuristic - A shortcut in decision-making that relies on the information that is most readily available, rather than the total body of information on a subject disconfirmation principle - the evidence obtained from testing demonstrated that the solution does not work.

Following an injury a patient has difficulty creating new long-term memories accompanied by drastic personality changes. Which of the following structures were most likely damaged by the injury? I. The hippocampus II. The thalamus III. The prefrontal cortex IV. The amygdala A. I and III B. II and III C. II and IV only D. I, II, and IV

Correct answer: A Hippocampus—Consolidates information to form long-term memories and can redistribute remote memories to the cerebral cortex Thalamus—A portion of the forebrain that serves as a relay and sorting station for sensory information, and then transmits the information to the cerebral cortex Prefrontal cortex—The anterior portion of the frontal lobes and is associated with planning intricate cognitive functions, expressing personality, and making decisions Amygdala—A structure that plays an important role in defensive and aggressive behaviors, including fear and rage *Injury to the prefrontal cortex can cause extreme personality changes (Phineas Gage is a classic example of this), and the hippocampus is responsible for creating long-term memories. The thalamus is primarily associated with sensory processing, and the amygdala is primarily associated with emotion. The correct answer is (A).

George says that he wouldn't cheat on the medical licensing exam because he's afraid of getting caught and not getting into medical school while Eric says he wouldn't cheat because it would lead to a decrease in the standard of care and he has an obligation to do what's best for everyone. According to Kohlberg, which of the following describes the phases of moral reasoning demonstrated by George and Eric, respectively? A. Preconventional; postconventional B. Conventional; preconventional C. Preconventional; conventional D. Conventional; postconventional

Correct answer: A Preconventional morality---->Preadolescence--->1: Obedience 2: Self-interest Conventional morality--->Adolescence to adulthood--->3: Conformity 4: Law and order Postconventional morality--->Adulthood (if at all) 5: Social contract 6: Universal human ethics In depth: The first of these phases, preconventional morality, is typical of preadolescent thinking and places an emphasis on the consequences of the moral choice. Stage one (obedience) is concerned with avoiding punishment (If I steal the drug, I'll go to jail) while stage two (self-interest) is about gaining rewards (I need to save my wife because I want to spend more of my life with her). Stage two is often called the instrumental relativist stage because it is based on the concepts of reciprocity and sharing: I'll scratch your back; you scratch mine. The second phase is conventional morality which begins to develop in early adolescence when individuals start to see themselves in terms of their relationships to others. This phase is based on understanding and accepting social rules. Stage three (conformity) places emphasis on the "good boy, nice girl" orientation in which a person seeks the approval of others (I should not steal the drug because stealing is wrong). Stage four (law and order) maintains the social order in the highest regard (If everyone stole things they couldn't afford, people who produce those items would not be able to continue their business). The third phase is postconventional morality which describes a level of reasoning of which not everyone is capable and is based on social mores which may conflict with laws. Stage five (social contract) views moral rules as conventions that are designed to ensure the greater good with reasoning focused on individual rights (Everyone has a right to live; businesses have a right to profit from their products). Finally, stage six (universal human ethics) reasons that decisions should be made in consideration of abstract principles (It is wrong for one person to hold another's life for ransom). In summary, only (A) is the correct answer. Step by step Simplify the question This question gives the example of two different people stating that they would not cheat on a medical licensing exam for different reasons. One would not cheat for self-interest and another would not because he believes the test is meant to ensure that he can take care of patients. One way of rewording the question would be: How does Kohlberg explain moral reasoning? According to Kohlberg, which of the following describes the phases of moral reasoning demonstrated by George (a student who won't cheat on a licensing exam due to self-interest) and Eric (a student who won't cheat on a licensing exam due to the interest of future patients), respectively? What is known? Unknown? Before we are able to analyze the scenario, we must first recall Kohlberg's phases of moral reasoning. His theory is based on the idea that as our cognitive abilities grow, we are able to think about the world in more complex and nuanced ways which directly affects the ways in which we resolve moral dilemmas and perceive the notion of right and wrong. Based on observations, Kohlberg organized moral thinking into three phases with two stages each. Preconventional morality—Typical of preadolescent thinking; places emphasis on consequences of moral choice; stages include concerns for avoiding punishment and gaining rewards Conventional morality—Begins to develop in early adolescence; based on understanding and accepting social rules; stages include placing emphasis on seeking approval of others and maintaining social order in highest regard Postconventional morality—Level of reasoning of which not everyone is capable; based on social mores which may conflict with laws; stages include viewing moral rules as conventions designed to ensure greater good with focus on individual rights and reasoning that decisions should be made in consideration of abstract principles The unknown in this question is how Kohlberg's phases of moral reasoning explain the two reasons for not cheating on the exam. What is your prediction? George's reasoning for not cheating is based on a fear of punishment for the action and a lack of receiving the reward (gaining acceptance into medical school). This is consistent with the preconventional phase which is focused on avoiding punishment and gaining rewards. Eric, however, will not cheat because he wants to ensure the greater good: that he does not undermine the standard of care provided to the public by doctors. This reasoning is consistent with the postconventional phase which views moral rules as a way to ensure the greater good. (A) is therefore correct.

Which process allows for the quick recognition of an object without analysis of the details of that object? A. Bottom-up processing B. Top-down processing C. Perceptual organization D. Continuity.

Correct answer: B Choice (B) . Top-down processing uses past experiences and memories to quickly identify an object. For example, if an individual is given a picture of a group of people, they would be able to identify their mother in the group much faster than if they were asked to identify a complete stranger.

According to the psychodynamic perspective, which of the following is likely to be the most influential in the moral reasoning of Kohlberg's preconventional phase? A. The id B. The ego C. The superego D. The libido

Correct answer: A Simplify the question The question stem seeks to find a similarity between the psychodynamic perspective and the moral reasoning of Kohlberg's preconventional phase. One way to reword the question would be: What part of the psychodynamic perspective represents the same moral reasoning as the preconventional phase of Kohlberg's theory? What is known? Unknown? Kohlberg's theory of moral development is divided into three phases; however, the question stem only asks for a comparison of the preconventional phase which occurs first. We do not therefore need to recall the other two phases. Preconventional morality—Typical of preadolescent thinking; places emphasis on consequences of moral choice; stages include concerns for avoiding punishment and gaining rewards Since the question asks for a comparison between the preconventional phase of Kohlberg's theory and the psychodynamic perspective, we must also recall Sigmund Freud's theory which contains three major entities as defined below. Id—Consists of all basic, primal, inborn urges to survive and reproduce; functions according to the pleasure principle which aims to achieve immediate gratification in order to relieve pent-up tension Ego—Takes into account objective reality as it guides or inhibits activity of id and id's pleasure principle as well as desires of superego; understood as organizer of mind Superego—Acts as personality's perfectionist by judging our actions and responding with pride at our accomplishments or guilt at our failures The unknown in this question is how the psychodynamic perspective relates to the preconventional phase of Kohlberg's theory of moral reasoning. More specifically, the question seeks to find similarities between the two. What is your prediction? According to Kohlberg's preconventional phase of moral reasoning, the individual is most concerned with personal consequences that include avoiding punishment or receiving a reward. Due to this, we could say moral reasoning during this phase is based on self-interest. Similarly, the id in the psychodynamic perspective utilizes the pleasure principle which seeks immediate gratification (another form of self-interest). (A) is therefore correct, because the id and the preconventional phase are both motivated by self-gratification.

A 35-year old woman has noticed an increased ability to converse about diverse topics intelligently even though her problem-solving ability has remained unchanged. Which of the following best describes this change? A. Her crystallized intelligence has increased while her fluid intelligence has remained the same. B. Her fluid intelligence has increased while her crystallized intelligence decreased. C. Her crystallized intelligence has increased while her fluid intelligence decreased. D. Her fluid intelligence has increased while her crystallized intelligence has remained the same.

Correct answer: A Simplify the question This question gives context about a woman whose ability to converse about diverse topics has increased while her problem- solving has remained the same. A scan of the answer choices indicates that it is asking specifically about how her fluid intelligence and crystallized intelligence have changed. Knowing this, the question could also be asked as: If a woman has noticed an increased ability to converse about diverse topics intelligently even though her problem-solving ability has remained unchanged, how has her fluid intelligence and crystallized intelligence changed if at all? What is known? Unknown? The definitions of crystalized vs. fluid intelligence are known. Crystallized intelligence—Related to use of learned skills and knowledge Fluid intelligence—Consists of problem-solving skills What is your prediction? If fluid intelligence is the capacity to solve problems based on experiences and this has remained unchanged for the patient, predict that this type of intelligence has remained the same. On the other hand, if her ability to carry out conversations on an array of topics has increased, predict that her crystallized intelligence has increased, because this is most concerned with learned skills. With this prediction, select (A).

Which of the following experimental proposals best explores the relationship between social media activity and social capital? A. Observing the level of engagement in political discussions on social media and the subsequent changes in online political group membership B. Measuring the amount of views a cooking video has on social media and subsequent changes in what participants buy at the grocery store C. Measuring time spent on social media and overall levels of happiness D. Observing the level of engagement in political discussion on social media and time spent exercising

Correct answer: A Social capital can be considered the investments that people make in their society in return for economic or collective rewards; the greater the investment, the higher the level of social integration and inclusion. One of the main forms of social capital is the social network. Social networks can create two types of social inequality: situational (socioeconomic advantage) and positional (based on how connected one is within a network and one's centrality within that network). It is claimed that inequality in networks creates and reinforces inequality in opportunities. Moreover, low social capital leads to greater social inequality. The question asks how social media activity can be related to the concept of social capital; hence, we are looking for a choice that ties social media activity to a person's social networking.The best match is (A) which ties online political discussions to online political group membership. While all the remaining answer choices mention social media, none of them tie it to social capital. Purchases at the grocery store (B), overall happiness (C), and time spent exercising (D) are all individual activities or characteristics and do not encompass how the participants network with other people Step by step Simplify the question This question addresses the relationship between the use of social media such as LinkedIn, Twitter, Instagram, and Facebook, and social capital. Specifically, we are asked which study would best explore the relationship between social media and social capital, so the question remains: Which of the following studies would best examine the relationship between social media and social capital? What is known? Unknown? We know the concept of social media because most of us use it on a daily basis to communicate with others in our respective networks; however, we need to revisit the definition of social capital. Social capital can be considered as the investments people make in their society in return for economic or collective rewards; the greater the investment, the higher the level of social integration and inclusion. One of the main forms of social capital is the social network. Social networks can create two types of social inequality: situational (socioeconomic advantage) and positional (based on how connected one is within a network and one's centrality within that network). To reiterate, a social network is a form of social capital. Similarly, when someone uses social media, someone is connected to a variety of people. These online connections may help us by sharing their resources with us. For example, someone may find a better job through LinkedIn connections. We now have to carefully examine all the answer options to find that particular study which would best explore the relationship between social media and social capital. What is your prediction? (A) describes both the online use of social media for political discussions as well as online political group membership. The latter can also be viewed as social capital because the group is one's social network. (A) therefore seems a promising answer; however, let's also review our other choices. Viewing a cooking video (B) is a social media activity but grocery shopping is typically a solitary activity; hence, social capital is excluded from this scenario and (B) is incorrect. (C) also involves social media but not social capital, because social capital is not directly associated with overall levels of happiness. Similarly, (D) also addresses social media but not social capital, because exercising can be considered a solitary activity. (A) is the only answer that addresses both social media and social capital, so it is therefore correct.

All of the following are functions that are primarily performed by the dominant hemisphere of the brain EXCEPT: A. music cognition. B. complex voluntary movement. C. arithmetic. D. language-related auditory processing.

Correct answer: A The dominant hemisphere (usually the left) is primarily analytic in function, making it well-suited for managing details. For instance, language, logic, and math skills are all located in the dominant hemisphere. Again, language production (Broca's area) and language comprehension (Wernicke's area) are primarily driven by the dominant hemisphere. The nondominant hemisphere (usually the right) is associated with intuition, creativity, music cognition, and spatial processing. The nondominant hemisphere simultaneously processes the pieces of a stimulus and assembles them into a holistic image. The nondominant hemisphere serves a less prominent role in language. It is more sensitive to the emotional tone of spoken language, and permits us to recognize others' moods based on visual and auditory cues, which adds to communication. The dominant hemisphere thus screens incoming language to analyze its content, and the nondominant hemisphere interprets it according to its emotional tone.

A genetically engineered mouse can have specific parts of its brain selectively inhibited through optogenetics. In this technique, when a probe-mounted light is shone onto the targeted region of the brain, that region is "turned off." If a light was shone onto the ventromedial hypothalamus, what behavior would be expected? A. The mouse will continually eat more food, despite having a full stomach B. The mouse will continually feel satiated, despite having an empty stomach C. The mouse will begin to display aggressive behavior towards its litter mates D. The mouse will begin to display prosocial behavior towards its litter mates

Correct answer: A The hypothalamus, subdivided into the lateral hypothalamus, ventromedial hypothalamus, and anterior hypothalamus. The lateral hypothalamus (LH) is referred to as the hunger center because it has special receptors thought to detect when the body needs more food or fluids. In other words, the LH triggers eating and drinking. When this part of the hypothalamus is destroyed in lab rats, they refuse to eat and drink and would starve to death if not force-fed through tubes. The ventromedial hypothalamus (VMH) is identified as the "satiety center," and provides signals to stop eating. Brain lesions to this area usually lead to obesity. Therefore, (A) is correct.

A firefighter holds his hand near a door handle to determine whether or not it's safe to touch when entering a burning house. Which of the following is responsible for transmitting that information? A. Afferent neurons B. Efferent neurons C. Cerebellum D. Reticular formation.

Correct answer: A The nervous system contains three types of neurons: sensory (also known as afferent neurons), motor (also known as efferent neurons), and interneurons. The sensory or afferent neurons (A) carry sensory information from receptors located in the peripheral nervous system to the spinal cord and brain.

How is the posterior pituitary different from the anterior pituitary? A. The posterior pituitary is comprised of axonal projections of the hypothalamus. B. The posterior pituitary releases FSH, LH, ACTH, TSH, prolactin, and GH. C. The posterior pituitary is not a part of the diencephalon. D. The posterior pituitary helps control circadian rhythms.

Correct answer: A The pituitary gland, sometimes referred to as the "master" gland, is located at the base of the brain, and is divided into two parts: anterior and posterior. It is the anterior pituitary that is the "master," because it releases hormones that regulate activities of endocrine glands; however, the anterior pituitary itself is controlled by the hypothalamus. The pituitary secretes various hormones into the bloodstream that travel to other endocrine glands located elsewhere in the body to activate them. Once activated by the pituitary, a given endocrine gland manufactures and secretes its own characteristic hormone into the bloodstream. The diencephalon also differentiates to form the posterior pituitary gland, pineal gland, and connecting pathways to other brain regions. The posterior pituitary is comprised of axonal projections from the hypothalamus and is the site of release for the hypothalamic hormones antidiuretic hormone (ADH, also called vasopressin) and oxytocin. The correct answer is (A). (B) is true of the anterior pituitary and not the posterior pituitary. (C) is false, the posterior pituitary is part of the diencephalon while the anterior pituitary is not. (D) is true of the pineal gland. Therefore, (A) is the only correct answer.

Which of the following informed consent situations is unethical? A. A patient who speaks only Spanish is admitted to the hospital for elective surgery. She has her doctor, who speaks only English, sign the consent for her after being unable to find a translator. B. A doctor in the emergency room signs a general treatment consenst for a patient who presents unconscious. C. A 14 year old girl with Down syndrome is found to have appendicitis requiring emergency surgery so her father signs the consent for her. D. A schizophrenic patient doing well on medication comes into the ambulatory surgery center for a medically indicated tonsillectomy and signs his own consent form.

Correct answer: A This question requires knowledge of the definition of informed consent, and what exceptions can be made. In the case of mental disability and emergency situations, informed consent is not needed. A language barrier is not a valid reason to allow for a proxy give consent on behalf of the patient. In the absence of an emergent condition, this decision most certainly cannot be made for the patient by the rendering physician. Thus, (A) is correct. *Informed consent means that a patient must be adequately counseled on the procedures, risks and benefits, and goals of a study to make a knowledgeable decision about whether or not to participate in the study. If a patient is not able to understand the language, they are not able to make informed consent.

If a human can detect a change in the noise level of a quiet room when the sound intensity is increased from 40 units to 44 units, then according to Weber's law, what increase in intensity is required to be detectable on a busy street with an ambient sound level of 80 units? A. (4/40) × 80 B. (44/40) × 80 C. (40/44) × 80 D. (44 − 40) × 80

Correct answer: A To solve this question, first recognize that the difference threshold or just-noticeable difference (jnd) is the minimum difference in magnitude between two stimuli before one can perceive this difference. Weber's law states that the jnd for a stimulus is proportional to the magnitude of the stimulus and that this proportion is constant over most of the range of possible stimuli. This is calculated by the equation Δ I I = k where Δ I is the change from I required to cause a difference. The calculated k represents the jnd for a specific case. In this scenario, a four- unit change is required for students to be able to hear a difference in sound, so 4/40 would equal k . From there, multiply 4/40 × 80 to see what the jnd would be for this specific situation. Only (A) is the correct answer. step by step Simplify the question This question wants to see if we can correctly apply Weber's law. Just remember that this law is proportional. One way to reword the question would be: How is Weber's law used to calculate how humans perceive changes in stimuli? What is known? Unknown? Weber's law addresses the ability of a human to perceive changes in stimuli. It applies to changes in all of the following: volume, pitch, brightness of a light, and even the weight of an object. It essentially says that the jnd, the amount of actual change that must occur in a stimulus before our minds can perceive the change, is proportional to the initial stimulus. We know that the proportion of change is the proportional difference: in this case, an increase of 10% going from 40 to 44. The unknown is how much change would be needed from a new baseline of 80 units, but Weber's law tells us that the proportion of change is constant, so we can calculate that the new change is 10% of 80 or 8 units. What is your prediction? When the MCAT gives a numerical question on Weber's law, it is wanting to see if we understand that Weber's law uses a constant proportion of change in stimuli. The question will give us enough information to calculate a constant proportion. We then need only apply that proportion to the third value given in the question to get the result. The question tells us that a four-unit change from a base of 40 units is perceivable. This is a 10% change (4/40 = 10%). This means that if the ambient sound level was 80 units, it would have to increase by the same 10% (10% × 80) or 8 units in order for us to perceive a difference. The only formula that gives the correct answer of 8 is (4/40) × 80 (A).

Which of the following is the correct order in which visual information reaches the cerebral cortex? A. Retina, optic nerve, optic chiasm, optic radiations, occipital lobe B. Retina, optic radiations, optic chiasm, optic nerve, occipital lobe C. Retina, optic chiasm, optic nerve, occipital lobe, optic radiations D. Retina, optic nerve, optic radiations, optic chiasm, occipital lobe.

Correct answer: A Visual information is picked up by the retina in the eye. Here, the information is passed to the optic nerve, crosses over at the optic chiasm, moves along the optic radiations to the occipital lobe of the cerebral cortex. Choice (A) correctly orders the pathways and thus is the correct answer.

Which of the following would affect the accuracy, but not the precision of a study conducted by researchers? A. Instruments that consistently read higher than actual values B. Instruments that read higher or lower than actual values randomly C. A wide range of instruments that either read consistently high or low D. An instrument that is accurate only when the mean of 5 trials is taken.

Correct answer: A Accuracy is the ability to measure a true value, precision is the ability to measure consistently over many trials. The only precise example in the answers, choice (A) .

A new resident is having difficulty keeping up with the amount of paperwork, pressure to publish case studies, and an increased patient load. However, he's still able to be what he considers a good father to his son. Which of the following is the resident experiencing? A.Role exit B. Role strain C. Role conflict D. Role partner

Correct answer: B Role—a set of beliefs, values, attitudes, and norms that define expectations for those who hold the status. For example, a medical doctor is expected to see patients, fill out paperwork, and publish case studies Role performance—how someone carries out a particular role. For example, a medical doctor has to translate medical terms in language that the patient can understand Role conflict—difficulty in satisfying the requirements or expectations of multiple roles. For example, a medical doctor has difficulty with satisfying work demands, being a parent and a husband. In other words, there is a conflict among different roles. Role strain—difficulty in satisfying multiple requirements of the same role. For example, a medical doctor has difficulty in keeping up with all the requirements of his work Role exit—someone drops one identity for another. For example, a medical doctor retires and becomes an avid fisherman Role partner—the person with whom one is interacting. For example, a medical doctor interacts with patients, nurses, other doctors, administrative staff and so forth

A young boy is very hungry after school and, having no money, considers taking a candy bar from a convenience store on the way home. However, he begins to feel guilty about the idea and decides to wait to eat until he gets home. Which of the following statements regarding this scenario is most accurate? A. The desire to steal food and eat originated from the id. The decision to wait originated from the ego. The feeling of guilt originated from the superego. B. The feeling of guilt originated from the id. The desire to steal food and eat originated from the ego. The decision to wait originated from the superego. C. The desire to steal food and eat originated from the id. The feeling of guilt originated from the ego. The decision to wait originated from the superego. D. The decision to wait originated from the id. The desire to steal food and eat originated from the ego. The feeling of guilt originated from the superego.

Correct answer: A Id—Consists of all the basic, primal, inborn urges to survive and reproduce and functions according to the pleasure principle which aims to achieve immediate gratification in order to relieve pent-up tension Ego—Takes into account objective reality as it guides or inhibits the activity of the id and the id's pleasure principle as well as the desires of the superego; it can be understood to be the organizer of the mind Superego—Acts as the personality's perfectionist by judging our actions and responding with pride at our accomplishments or guilt at our failures

For a unimodal distribution with a negative skew, what are the measures of central tendency from least to greatest? A. Mean, mode, median B. Mean, median, mode C. Mode, median, mean D. Median, mode, mean

Correct answer: B A normal distribution, is symmetrical in which values are evenly distributed around a central mean, which also equals the median and mode. It approximates a bell curve when plotted on a graph. bell curve with probability on the y-axis and mean and standard deviations on the x-axis A skewed distribution is one that contains a tail on one side or the other of the data set. A negatively skewed distribution has a tail on the left (or negative) side, whereas a positively skewed distribution has a tail on the right (or positive) side. negative skew has tail on left, mean < median < mode; positive skew has tail on right, mode < median < mean Because the mean, or average of a set of data is calculated by adding up all of the individual values within the data set and dividing the result by the number of values, it is the most susceptible by outliers. Having an outlier—an extremely large or extremely small value compared to the other data values—can shift the mean toward one end of the range. For example, the average income in the United States is about $70,000, but half of the population makes less than $50,000. In this case, the small number of extremely high-income individuals in the distribution shifts the mean to the high end of the range. The median value for a set of data is its midpoint, where half of data points are greater than the value and half are smaller. The median tends to be the least susceptible to outliers, but may not be useful for data sets with very large ranges or multiple modes. The mode is the number that appears the most often in a set of data. When we examine distributions, the peaks represent modes. The mode is not typically used as a measure of central tendency for a set of data, but the number of modes, and their distance from one another, is often informative. In all, due to the mean being more susceptible to outliers than the median, the mean will be lower than the median. The mode is least susceptible and will be greatest (B).

A student feels embarrassed following a poor performance at a piano recital. The next day, he vows to practice the piano for an hour every day. This is an example of: A. escape learning. B. avoidance learning. C. positive reinforcement. D. negative punishment.

Correct answer: B Avoidance learning: Negative reinforcers can be subdivided into escape learning and avoidance learning, which differ in the timing of the unpleasant stimulus. Taking aspirin is an example of escape learning: the role of the behavior is to reduce the unpleasantness of something that already exists, like a headache. Avoidance learning, on the other hand, is meant to prevent the unpleasantness of something that has yet to happen. In fact, you are practicing avoidance right now: you are studying to avoid the unpleasant consequence of a poor score on the MCAT. When you do well on Test Day, that success will positively reinforce the behavior of studying for the next major exam of your medical career: the USMLE! (B) is correct.

A man buys a new set of speakers for his television. Before he has a chance to connect them, his friend walks into the room and remarks that the sound quality coming from the television is vastly improved. Which of the following concepts best explains his friend's mistake? A. Difference thresholds B. Signal detection theory C. Correct negative D. Selective attention

Correct answer: B Because the speakers were not yet connected, it was likely the friend's expectations that led him to remark on the improved sound quality. Signal detection theory helps to explain how elements of context such as experiences, emotions, motives, and, as in this example, expectations can shape our perceptions to identical stimuli. (B) is therefore the best answer. This is not an example of a difference threshold (A) which refers to the perception of an actual change in the stimulus in question, so (A) is incorrect. A correct negative (C) occurs when the subject correctly identifies that no signal, or in this case no change in signal, was given. The friend's response would rather be termed a false alarm in which the subject seems to perceive a signal when none was given. Correct negatives and false alarms are a part of signal detection theory which explains how our perception of the same stimuli changes depending on both internal and external context. (C) is incorrect. Selective attention (D) describes the tendency to focus on certain stimuli while filtering out others. This does not help explain why he perceived a change in sound quality when in fact such a change did not exist. (D) is incorrect. The concept of signal detection theory focuses on the changes in our perception of the same stimuli depending on both internal (psychological) and external (environmental) context. For example, how loud would someone need to yell your name in a crowd to get your attention? Part of the answer comes from psychology: if you heard something that sounds vaguely like your name, would you likely acknowledge it or not? The answer is not merely a yes or no, but would depend on the size of the crowd; your expectation of being called; social factors such as the makeup of the crowd and your comfort with the other individuals; and personality. Highly sociable, extroverted individuals tend to hear their name more easily than quieter, introverted individuals. Only (B) is the correct answer. Signal detection theory—The effects of nonsensory factors such as experiences, motives, and expectations on the perception of stimuli.

A child has a self-centered view of the world and has difficulty understanding that others may have thoughts and feelings different from their own. According to Piaget, the child is most likely in which stage of cognitive development? A. Sensorimotor B. Preoperational C. Concrete operational D. Formal Operational.

Correct answer: B Choice (B) . All of the answer choices represent the four stages of cognitive development according to Piaget's model. The preoperational stage of development (approximately 2 years to 7 years of age) is characterized by symbolic thinking, egocentrism, and centration. The question stem describes egocentrism, or the inability to imagine what another person may think or feel, and (B) is thus correct.

According to the Yerkes-Dodson law, which of the following is true about task difficulty and arousal? A. For simple tasks, low arousal is associated with peak performance B. For simple tasks, high arousal is associated with peak performance C. For complex tasks, low arousal is associated with peak performance D. For complex tasks, high arousal is associated with peak performance.

Correct answer: B Choice (B) . The Yerkes-Dodson law of social facilitation states that simpler tasks are performed better with higher amounts of arousal. For example, if a person is told to press a button continuously for an hour, the task would be completed more effectively if the person is allowed to listen to music than if the person was forced to sit in complete silence. Furthermore, this increase in performance is not seen if the task is too complex since complex tasks require more focus and high levels of arousal may actually distract the individual from completing the task effectively.Choice (C) and (D) . Peak performance on complex tasks is predicted with moderate arousal.

An adult psychiatric patient with schizophrenia believes that the singer of a popular song on the radio is talking directly to him. This is an example of: A. a delusion of persecution. B. a delusion of reference. C. a delusion of grandeur. D. catatonia.

Correct answer: B Delusions of reference are one type of positive symptom commonly seen in patients with schizophrenia. Delusions are false beliefs discordant with reality and not shared by others in the individual's culture that are maintained in spite of strong evidence to the contrary. Delusions of persecution, which involve the belief that the person is being deliberately interfered with, discriminated against, plotted against, or threatened. The question stem does not imply that the person perceives the singer to be threatening or speaking badly about him, allowing (A) to be eliminated. Delusions of grandeur, also common in bipolar I disorder, involve the belief that the person is remarkable in some significant way, such as being an inventor, historical figure, or religious icon. Since the patient is more focused on the singer, rather than his own self worth, (C) can be eliminated. Other common delusions involve the concept of thought broadcasting, which is the belief that one's thoughts are broadcast directly from one's head to the external world, and thought insertion, the belief that thoughts are being placed in one's head. Catatonia refers to the large reduction in a patient's spontaneous movement and activity. The patient may maintain a rigid posture, refusing to be moved. Catatonic behavior may include useless and bizarre movements not caused by any external stimuli, echolalia (repeating another's words), or echopraxia (imitating another's actions). Since the patient's activity level did not decrease, based on the question stem, (D) can be eliminated. The patient in this question stem believes that the singer is talking to him directly through the radio, even though the likelihood of that actually happening is very low. This is an example of a delusion of reference, which involves the belief that common elements in the environment are directed toward the individual. In this case, the common elements in the environment are the song lyrics on the radio. Thus, (B) is correct.

The model of emotional expression which posits that biologically predetermined expressions of emotion are preceded by cognitive assessments of the situation is the: A. social construction model. B. appraisal model. C. basic model. D. psychological construction model.

Correct answer: B In the appraisal model of emotional expression (B), cognitive processes occur in response to a situation that then triggers an emotion followed by biologically predetermined expressions of that emotion, so (B) is the correct answer. The social construction model (A) assumes that there is no biological basis for emotions. Instead, emotions are based on experiences and the situational context alone. Because the social construction model rejects the role of biology, it does not match the model described in the question stem. It also suggests that certain emotions can only exist within social encounters and that emotions are expressed differently—and thus play different roles—across cultures. According to this model, one must be familiar with social norms pertaining to a certain emotion to perform the corresponding emotional behavior(s) in a given social situation. The basic model of emotional expression (C) was first established by Charles Darwin. Darwin emphasized the biologically predetermined expressions of emotion: facial expressions, behaviors, postures, vocal changes, and physiological changes. The basic model does not include a cognitive antecedent to the physical expression of emotion, so (C) is incorrect. The psychological construction model (D) explains emotion as a manifestation of basic psychological processes, so it is incorrect as well; hence, only (B) is the correct answer.

A researcher wants to study what, if any, barriers to safe sex exist among men in the U.S. who have sex with other men (MSM). The researcher proposes to conduct in-depth interviews, and will recruit participants from gay bars in several major metropolitan areas. Which of the following is the biggest design flaw in the researcher's proposed study? A. In-depth interviews are not appropriate for this proposed topic of study. B. The population of men in gay bars may be substantially different from the overall population of men who have sex with men. C. The cities from which the researcher will recruit may not be representative of cities in other countries around the world. D. Men who have sex with men may not practice safe sex.

Correct answer: B In this case, the researcher is interested in any barriers to safe sex among MSM. The population studied is defined based on sexual behavior (sex with men) rather than a sexual identity (gay men). Many more men have sex with men than who identify as gay and would socialize in gay bars. Therefore, the researcher's idea to recruit solely from gay bars excludes a large number of men who have sex with men from inclusion in the study, and this group most likely differs in meaningful ways from gay men in general. (B) is correct. In-depth interviews, discussed in (A), are perfectly acceptable in this case; there may be other methodologies that are also acceptable, but this is not a major flaw. Given that this particular population is relatively small and may be considered a hidden population, traditional quantitative research methods may not be appropriate or possible. The researcher was interested in the existence of barriers to safe sex among men in the United States. Therefore, (C) is incorrect. (D) is not a flaw, but is actually one of the things the researcher is trying study. (B) is the only correct answer.

An individual with a high internal locus of control would be most likely to agree with which of the following statements? A. "I am a brilliant, kind, and generous person" B. "I am the master of my own destiny" C. "In the end, everything works out the way it should" D. "The world is a just and fair place".

Correct answer: B Individuals with an internal locus of control view themselves as having significant, direct control over their lives, while those with an external locus of control believe that events in their life result from external forces beyond their control. Thus choice (B) reflects the belief of an individual with a high internal locus of control.

When neurological symptoms such as numbness or incontinence reveal no underlying physiological cause, a plausible psychological explanation would be: A. paranoid personality disorder. B. conversion disorder. C. body dysmorphic disorder. D. Parkinson's disease.

Correct answer: B Paranoid personality disorder—A disorder marked by a pervasive distrust of others and suspicion regarding their motives Conversion disorder—A disorder characterized by unexplained symptoms affecting voluntary motor or sensory functions Body dysmorphic disorder—A disorder characterized by an unrealistic negative evaluation of one's appearance or a specific body part Parkinson's disease—A disease associated with a loss of dopaminergic neurons in the basal ganglia. These disruptions of dopamine transmission lead to resting tremors and jerky movements, as well as postural instability.

How is the mean of a data set calculated? A. Find the number that is in the middle when the numbers are arranged from least to greatest B. Find the number that occurs most often in the data set C. Find the sum of all of the numbers in the data set and then divide by the number of entities in the set D. Determine the entity in the data set that best summarizes the data.The mean is another way of saying the average.

Correct answer: C This means that it is calculated by taking the sum of all of the numbers and dividing by the amount of numbers. This matches choice (C) .

Which of the following statements about the idea of the self-concept is true? A. A person has multiple self-concepts that can change depending on the context and often compete for expression. B. Each of us has one self-concept that describes who we are, but we have multiple identities that connect us to specific groups. C. Self-schemata can pertain to a person's past and future, while the self-concept is limited to the present. D. The self-concept includes unconscious elements and represents a battle between the conscious and unconscious.

Correct answer: B Self-concept—Formed from our own internal list of answers to the question "Who am I?" and includes our appraisal of who we used to be and who we will become: our past and future selves Self-schema—A self-given label that carries with it a set of qualities. For example, athlete self-schema usually carries the qualities of youth, physical fitness, and dressing and acting in certain ways Identity—The individual components of our self-concept related to the groups to which we belong

All of the following scenarios illustrate the concept of signal detection theory EXCEPT: A. a woman at the beach ignores a party of strangers calling her name. B. a man walking through a carnival participates in a game every time someone asks. C. a woman walks past a perfume stand in the mall even though an employee calls to her. D. a man who is waiting for friends at a concert turns when he hears his name.

Correct answer: B Signal Detection Theory—The concept that our perception of a stimulus changes depending on our experiences, motives and expectations

"Arithmetic," when used as a noun, is typically pronounced with emphasis on the second syllable. However, when used an adjective, the emphasis is typically placed on the third syllable. This can be characterized as a distinction of which component of speech? A. Semantics B. Phonology C. Morphology D. Syntax

Correct answer: B The correct answer is (B). Phonology refers to the actual sound of language. In the example if the question stem, one "phoneme," or speech sound, is exchanged for another, depending on the usage of the word. There are about 40 speech sounds or phonemes in English, although many more exist in other language. Semantics refers to the association of meaning with a particular word. Morphology refers to structure of words. Finally, syntax refers to how words are put together in sentences.

Which of the following scenarios best demonstrates the iron law of oligarchy? A. A hospital is made more efficient by limiting the amount of time patients are seen by doctors. B. A hospital executive board is created with different types of stakeholders, but ends up being controlled by a few wealthy donors. C. A hospital is run by non-elected, paid officials that make more money as they achieve set milestones. D. A hospital is set up to attain specific monetary and patient-care goals and quickly adopts a structure and culture once it is open.

Correct answer: B The iron law of oligarchy states that democratic or bureaucratic systems naturally shift to being ruled by an elite group. That means that the power is in the hands of just a few people.

Given the following data set, what is the mode?3, 2, 5, 5, 1, 6, 6, 7, 7, 3, 4, 4, 1, 3, 4, 5, 6, 6, 1 A. 5 B. 6 C. 4.21 D. 2.

Correct answer: B The mode of a data set is the most frequent number that occurs. Here "6" occurs four times and is thus the mode of the data set, or choice (B) .

A student is accepted to a prestigious high school, but cannot attend because the bussing system does not provide transportation to and from his neighborhood. This is best described as which type of social inequality? A. Environmental injustice B. Spatial inequality C. Social immobility D. Geographical stratification

Correct answer: B The student has encountered a physical barrier to his access of institutional resources, so this would be defined as a form of spatial inequality (B). Spatial inequality is a form of social stratification across territories and their populations, and can occur along residential, environmental, and global lines. Environmental injustice (A) refers to an uneven distribution of environmental hazards in communities. Lower-income neighborhoods may lack the social and political clout to prevent the placement of environmental hazards in their neighborhoods. However, environmental hazards are not discussed in the question stem, making this choice incorrect. Social immobility (C) refers to the inability to move in the social hierarchy through changes in income, education, occupation, etc. This is not what is directly implied in the question stem. Although there is a difference in access to transportation based on geography, the passage does not provide information about the socioeconomic status of people living in the student's neighborhood or of people living in the neighborhood of the school. Therefore, (D) cannot be inferred.

Researchers studying the relationship between patients' socioeconomic status (SES) and their risk of developing lung cancer conducted a year-long study that followed a thousand patients in zip codes associated with a low SES and another thousand in zip codes associated with a higher SES. This study design is an example of: A. cross-sectional study design. B. cohort study design. C. case-control study design. D. experimental study design.

Correct answer: B This question requires matching the example study in the question to the appropriate type of study. The question describes a study wherein the subjects are separated into groups (zip codes), and are assessed throughout time for a certain outcome (lung cancer). This is an example of a cohort study (B). A cross-sectional study typically involves categorizing patients into different groups at a single point in time to assess for prevalence. The question shows an example of a cohort study, which follows a group and assesses them at different points in time. A cohort study follows a group and assesses them at different points in time. A case-control study is one wherein subjects with a certain outcome are assessed for previous risk factors. The question shows an example of a cohort study, which follows a group and assesses them at different points in time. An experimental study involves separating a group into two randomized groups in order to see the effects of a certain variable on a sample. The question shows an example of a cohort study, which follows a group and assesses them at different points in time.

Political parties are sometimes guilty of focusing solely on ideas generated by members within their respective parties, while ignoring and dismissing the opposing party's views. This is most clearly an example of: A. deindividuation. B. groupthink. C. peer pressure. D. group polarization.

Correct answer: B Choice (B) . Groupthink occurs when individuals in a group ignore outside ideas and thoughts and focus only on member ideas. This phenomenon leads to members not considering all points of the view and dissenting opinions unable to express their beliefs.Choice (D) refers to the phenomenon whereby groups tend to make decisions that are more extreme than the initial inclinations of the members. Since the question refers to the dismissal of new ideas by the group as a whole and nothing is known of the members initial inclinations, group polarization does not accurately explain the observation.

Which sociological theory would view crime as a disruption to the equilibrium of a social order? A. Social constructionism B. Conflict theory C. Functionalism D. Symbolic interactionism

Correct answer: C (A) focuses on how individuals put together their social reality by communicating and working together to agree on the significance of a concept or principle. Crime would therefore be understood as a social construct defined by the interactions, opinions, and values of the members of a society. (B) focuses on how power differentials are created and how these differentials contribute to the maintenance of social order. A conflict theorist might point out the ways in which imbalances in economic, political, and social resources contribute to crime. (D) emphasizes the ways individuals interact through a shared understanding of words, gestures, and other symbols, including the role of certain behaviors. Differences in the way we understand certain symbols lead to instances of societal discord, such as crime. Only (C), the theory of functionalism, focuses on the way that individuals' actions contribute to the harmony, or equilibrium, of society. According to these theorists, functions help keep society in balance. In contrast, dysfunctions are harmful consequences of people's actions as they undermine a social system's equilibrium. This theory matches the one described in the question, so (C) is the correct answer.

Synsepalum dulcificum is a plant that produces a berry known as the miracle berry because it makes sour foods taste sweet. Which of the following could explain how this occurs? I. The miracle berry blocks sour taste receptors in the presence of sour foods. II. The miracle berry activates sweet receptors in the presence of sour foods. III. The miracle berry has an active compound that acts directly on the taste center in the brain. IV. The miracle berry only interacts with the portion of the tongue containing taste buds for sweet foods. A. I only B. II and IV only C. I and II only D. I, II, III and IV

Correct answer: C As a sense, taste is often simpler than we imagine. There are five basic tastes: sweet, sour, salty, bitter, and umami (savory). Tastes are detected by chemoreceptors; however, unlike olfactory chemoreceptors, taste chemoreceptors are sensitive to dissolved compounds. Sourness is a reaction to acid such as lemon juice or vinegar. Sweet, bitter, and savory flavors are also triggered by specific molecules binding to receptors. The receptors for taste are groups of cells called taste buds which are found in little bumps on the tongue called papillae. Taste information travels from taste buds to the brainstem and then ascends to the taste center in the thalamus before traveling to higher-order brain regions. In depth The compound in the miracle berry could plausibly interact with taste receptors as described in Statements (I) and (II). The blood- brain barrier would likely prevent the direct interaction described in Statement (III). Interestingly, Statement (IV) is based on a common but scientifically inaccurate misconception that different areas of the tongue contain different kinds of taste receptors. The correct answer is (C) Step by step Simplify the question This miracle berry must have some physiological effect that changes the taste of sour to the taste of sweet. Another way to ask the question is: What would have to happen in the body in order to mask a sour taste and replace it with a sweet taste? What is known? Unknown? To answer this question, we need to think about how our bodies interpret taste. If we eat something sweet, the chemoreceptors in our tongue that are triggered by sweet compounds are activated, so in order to experience a flavor as sweet, those chemoreceptors need to activate. Similarly, if the chemoreceptors that are triggered by sour, acidic compounds activate, we will experience a sour taste. What is your prediction? Notice that the question is asking how the miracle berry would make not just any food taste sweet, but would specifically make sour foods taste sweet. In order for this to happen, it would have to simultaneously activate the sweet chemoreceptors and deactivate the sour chemoreceptors which would mean that Statements (I) and (II) both have to be true. (C) is th

Which of the following psychologists developed a systematic 3-stage, 6-level sequence to explain moral development? A. Sigmund Freud B. Erik Erikson C. Lawrence Kohlberg D. Lev Vygotsky.

Correct answer: C Choice (C) . All four choices are psychologists who were influential in the area of personality development. However, Lawrence Kohlberg was responsible for introducing a theory of personality development that focused on moral reasoning. Kohlberg believed that individuals progress through six stages of moral development, with each stage signifying more developed and advanced arguments/reasoning of moral dilemmas.

All of the following are characteristic of Alzheimer's disease EXCEPT: A. diffuse cerebral atrophy. B. senile plaques. C. decreased dopamine in the basal ganglia. D. reduced levels of acetylcholine..

Correct answer: C Choice (C) . All of the choices are found in alzheimer's disease except decreased dopamine in the basal ganglia. Decreased dopamine levels in the basal ganglia is associated with Parkinson's disease.

A researcher argues that individuals are aroused by physiological needs such as food, water, and shelter and that motivation comes from an attempt to satisfy those desires. Which of the following theories best describes the researcher's point of view? A. Behaviorism B. Arousal theory C. Drive reduction theory D. Incentive theory.

Correct answer: C Choice (C) . Drive reduction theory states that individuals experience internal tension that motivates them to take action to reduce the tension. Thus all behavior is internally motivated. The researcher in the question stem who believes that individuals are motivated to action by satisfying desires is representative of drive reduction theory.

Which of the following structures of the limbic system is responsible for creating long-term memories? A. Thalamus B. Hypothalamus C. Hippocampus D. Hippopotamus.

Correct answer: C Choice (C) . The hippocampus, located within the temporal lobe, is primarily responsible for creating long-term memories. This brain structure plays an important role in memory and learning.

To some, it is considered improper for a male physician to examine a female without her husband or father present. A physician is thus presented with a dilemma when the immediacy of a procedure conflicts with the patient's beliefs. This is an example of which of the following? A. Rituals B. Conflict theory C. Cultural barrier D. Nonmaleficence.

Correct answer: C Choice (C) . The physician is presented with a dilemma since his patient subscribes to cultural beliefs that are different from what the physician typically practices. Thus, the physician and patient are encountering a cultural barrier, or a social difference that is impacting the interaction between the two.Choice (A) . A ritual refers to a formal ceremony performed within a culture. Since the question describes a practice, or belief, this answer is incorrect.Choice (B) . Conflict theory emphasizes the importance of power and inequality within society, which does not describe the situation in the question stem.Choice (D) . Nonmaleficence is a ethical tenet of American medicine, but refers to the responsibility of healthcare worker to avoid treatments that pose a bigger risk to the patient than benefit.

A researcher observing the interactions between a mother and her child, notes that the child does not react to his mother entering or leaving the room. Furthermore, the child does not display any preference between his mother and the research assistant. These observations are best explained by concepts of which social behavior? A. Attraction B. Attachment C. Aggression D. Altruism.

Correct answer:B An attachment is an emotional tie with another person. This tie is shown in young children seeking closeness to their caregiver and showing distress upon separation. Thus, the best explanation for the child's behavior would require analysis of attachment, and (B) is the best answer.

Which of the following best describes the Schachter-Singer theory of emotion? A. Physiological arousal → emotion B. Physiological arousal + emotion → action C. Physiological arousal + cognitive appraisal → emotion D. Emotion + cognitive appraisal → action.

Correct answer: C Emotion is the combination of the physiological, behavioral, and cognitive responses to a situation. One of the hallmarks of the Schachter-Singer theories is that the contextual information of the environment affects the perception of emotion. For example, an individual's heart rate and respiratory rate may be interpreted as fear in the presence of a snake, but excitement when in the presence of an attractive individual. Thus, (C) is the correct answer.Choice (A) and (B) . These answers neglect environmental awareness.Choice (D) . The Schachter-Singer theory seeks to explain emotion as a result of physiological arousal and environmental awareness, so this is incorrect.

A physician testing for nerve damage in a diabetic patient places a vibrating pin on different parts of the patient's legs and feet. Which sensory receptors would be responsible for detecting this vibration? A. Nociceptors B. Baroreceptors C. Somatoceptors D. Osmoreceptors

Correct answer: C Exteroceptors—Receive and respond to stimuli originating from outside the body Interoceptors—Receive and respond to stimuli originating from within the body Proprioceptors—Detects motion and position of the body or a limb by responding to stimuli originating from within the body Somatoceptors—Applies to proprioceptors and exteroceptors collectively Classification based on what they detect: Nociceptors—Detect pain Osmoreceptors—Detect changes in the osmolarity of the blood Thermoreceptors—Detect changes in temperature Baroreceptors—Detect changes in blood pressure Mechanoreceptors—Detect touch and pressure Proprioceptors—Detect position Chemoreceptors—Detect chemical stimuli Electroreceptors—Detect electrofields Photoreceptors—Detect visible light Hydroreceptors—Detect humidity Magnetoreceptors—Detect magnetic fields

A researcher is looking to decrease the standard deviation of a sample taken from a population that is normally distributed. Which of the following changes to the experimental protocol could she make to accomplish this? A. Increase the interquartile range B. Decrease the interquartile range C. Increase the sample size D. Decrease the sample size

Correct answer: C Increasing the sample size increases the statistical power of the study, which reduces the sample variance and thereby the standard deviation. (C) is correct.

Which of the following is LEAST likely to be considered a symbol? A. A colleague saying "hello" B. The word "exit" on a road sign C. The grammatical structure of a complex sentence D. Exchanging vows at a wedding

Correct answer: C Language as a whole is a symbol, as it is a set of written markings or sounds with assigned meaning. The specific structure of a complex sentence in the English language is an example of grammar. While it is a part of language, grammar itself is not a symbol because it bears no cultural meaning on its own, meaning (C) is the correct answer. The exchanging of vows at a wedding is a ritual, or a formalized ceremony that often involves symbolism, and vows can be symbols themselves. Again, (C) is the only correct.

The function of which of the following brain structures in relation to Wernicke's and Broca's area is most analogous to the corpus callosum in relation to the left and right hemispheres of the brain? A. Optic chiasm B. Thalamus C. Arcuate fasciculus D. Primary auditory cortex

Correct answer: C Optic chiasm: As signal travels from the retina through the optic nerves toward the brain, the fibers from the nasal half (closer to the nose) of each retina cross paths at the optic chiasm. Thalamus: The thalamus functions as a preliminary sensory processing station and routes information to the cortex and other appropriate areas of the brain. Arcuate fasciculus: Broca's area and Wernicke's area are connected via the arcuate fasciculus, a bundle of axons which allows appropriate association between language comprehension and speech production. Primary auditory cortex: The auditory cortex is the primary site of most sound processing, including speech, music, and other sound information.

Prisons, when regarded as institutions of rehabilitation, can best be described as serving which of the following social-psychological functions? A. Primary socialization B. Secondary socialization C. Resocialization D. Anticipatory socialization

Correct answer: C Primary socialization—Occurs during childhood when we initially learn acceptable actions and attitudes in our society Secondary socialization—Occurs during adolescence and adulthood as we make smaller changes and refinements to behavior that were established in primary socialization to fit in with our peers at school, work, etc. that may have their own social rules Resocialization—Occurs at any point in life when we need to relearn acceptable actions and attitudes and discard old, socially-unacceptable behaviors Anticipatory socialization—Preparing for future changes in occupations, living situations, or relationships by learning the societal rules that will be expected of you in your new role

A director of a farmers' market hopes to optimize the market's efficiency. He assigns each customer a unique number, and instructs his cashiers to write down the number and the time of purchase for each sale they make. The director compiles the data and builds a map of the customer traffic. Which of the following describes the method the director is using to accomplish his goal? A. Semantic network B. Global Migration Group C. Network analysis D. Clusters of differentiation

Correct answer: C Semantic network: relate to how concepts are mapped in one's mind. The network in the question stem is a physical map, not one of the mind. When one node of our semantic network is activated, the other linked concepts around it are also unconsciously activated— a process known as spreading activation. For example, the color red may lead to an association with roses and fire engines. Global Migration Group: This is a United Nations group that deals with migration issues. It is not referred to in the question. Network analysis: The director is having the cashiers write down the number of customers at each store in an effort to represent customer traffic, which fits the definition of a network analysis. Clusters of differentiation: This is a process used to identified cell surface markers and would not be considered correct. Network—the observable pattern of social relationships among individuals or groups.

Which of the following studies best explores the mechanisms of social reproduction? A. A study observes the formation of a new ethnic enclave to explore if the social norms of the group will remain in their culture. B. Fertility rate is measured in communities of varying wealth. C. A study observes one group of poverty-level families who move to a low-poverty neighborhood while the control group remains in a high-poverty area. D. High school students who move to new districts and make new friends are observed to see if they adopt the habits and styles of their peers.

Correct answer: C Social reproduction is the proposition that social inequality, especially poverty, can be reproduced or passed on from one generation to the next; however, there are many other factors that contribute to poverty including where one lives and an emphasis on present-orientation in which people do not plan for the future. As a result, a component of social reproduction is that children of poor families are often born into poor neighborhoods or tough environments which makes it difficult for them to experience upward social mobility and escape poverty. (C) is the correct answer because it explores whether moving children from poverty-level neighborhoods to wealthier neighborhoods will affect the children's future financial success which can break the cycle of social reproduction. As previously mentioned, social reproduction has to do with social inequality rather than social norms.The introduction of a new ethnic enclave would only affect social reproduction if it caused existing groups to experience an increase or decrease in social status; therefore, because (A) mentions social norms rather than social inequalities, it is incorrect. (B) can be eliminated because social reproduction is not dependent on the fertility rate although a correlation may exist between fertility rate and social status. Rather, it explores whether offspring, regardless of how many there are, find themselves in the same social class as their parents once they have become adults. (D) is wrong because it is not about studying groups with different social statuses. Instead, examining whether students adopt the styles and habits of their peers would be an ideal experiment to explore the mechanisms of conformity; thus, only (C) is correct.

A country has new contraceptive programs and many women's rights groups begin to form. Which stage of demographic transition is this country most likely entering? A. Stage 1 B. Stage 2 C. Stage 3 D. Stage 4

Correct answer: C Stage 3 (C) occurs when birth rates finally decrease; this is often because there are improvements in contraceptives, more rights for women, and children requiring higher levels of education need more support from their families. Stage 1 (A) is categorized by high birth rates and high mortality rates; the population is stabilized because high mortality rates counteract the high birth rates. Stage 2 (B) is characterized by high birth rates but low mortality rates often due to the introduction of modern medicine. There is often rapid population growth in Stage 2, and many developing countries today are currently in this stage. Stage 4 (D) is characterized by low mortality and birth rates which result in a stabilized population growth. The correct answer is (C). Stage 1: Preindustrial society; birth and death rates are both high • Stage 2: Improvements in healthcare, nutrition, sanitation, and wages cause death rates to drop. • Stage 3: Improvements in contraception, women's rights, and a shift from an agricultural to an industrial economy cause birth rates (births per 1000 individuals per year) to drop. Furthermore, with an industrializing society, children must go to school for many years to be productive in society and may need to be supported by parents for a longer period of time than was formerly the case; families thus have fewer children. • Stage 4: An industrialized society; birth and death rates are both low

Why do doctors test primitive reflexes in infants? I. To conveniently check for neurological disorders. II. Reflexes can mark milestones for proper development. III. The reflexes generally disappear on a predictable schedule. IV. The Moro reflex supports an arboreal ancestry. A. I and II only. B. I and III only. C. I, II, and III only. D. I, II, III, and IV.

Correct answer: C The Moro Reflex While motor and startle reflexes exist in adults, infants have a number of primitive reflexes that disappear with age (III is true). For example, the rooting reflex is the automatic turning of the head in the direction of a stimulus that touches the cheek—such as a nipple during feeding. Sucking and swallowing when an object is placed in the mouth are also examples of reflexes related to feeding. Other primitive reflexes may have served an adaptive purpose in earlier stages of human evolution, but are currently used mainly in assessing infant neurological development. By comparing the point in time at which each of these reflexes disappears relative to the established norms, it is possible to tell whether neurological development is taking place in a normal fashion, making statements I and II true. The Moro reflex is when infants react to abrupt movements of their heads by flinging out their arms, then slowly retracting their arms and crying. It has been speculated that this reflex may have developed during a time when our prehuman ancestors lived in trees and falling could have been prevented by instinctive clutching. The Moro reflex usually disappears after four months and its continuation at one year is a strong suggestion of developmental difficulties. Asymmetry of the Moro reflex may hint at underlying neuromuscular problems. Look out for "faulty uses of detail" on the MCAT. Just because IV is a true statement does not mean it answers the question at hand, why doctors currently test primitive reflexes. Eliminating statement IV makes (C) correct.

Which of the following is best classified as controlled processing? A. An infant learning a word for the first time B. A man humming along to a catchy tune C. Consulting an English-Spanish dictionary D. Recognizing the voice of a friend in a crowded market

Correct answer: C The active memorization of information is known as controlled, or effortful, processing. By contrast, information that is gained without effort is said to be the result of automatic processing. Because (A), (B), and (D) are instances of passive acquisition of information, they are best described as automatic processing. Only (C) suggests an active form of active memorization, so it is the correct answer.

Which of the following most clearly illustrates the social component of the biopsychosocial approach in relation to a patient with schizophrenia? A. The patient's parents both suffered from schizophrenia B. The patient feels depressed that it's hard to find a job C. The patient has trouble affording medication due to being unemployed D. The patient had a brain scan that depicted excess dopamine in the brain.

Correct answer: C The biopsychosocial model takes into account everything that may affect a patient's illness and treatment, going beyond the illness itself. The bio aspect stresses the conventional biomedical portion; the fact that schizophrenia has a heritable component (A) and that it's associated with increased levels of dopamine in the brain (D) . The psycho aspect relates to how a patient feels about the disease and the consequences of having the disease (B) . Finally the social aspect deals with stressors the disease may have on the patient's family, friends, or career (C) .

Damage to which region of the central nervous system is most likely to cause hearing disturbances and a loss of pupillary reaction? A. Spinal cord. B. Hypothalamus. C. Midbrain. D. Hindbrain.

Correct answer: C The spinal cord primarily acts to move information from sensory neurons, to the brain, and back to motor neurons. Since hearing and vision organs are near the brain, they are unlikely to require spinal input and communicate with the brain directly instead. The hypothalamus's primary functions are to regulate homeostasis and integrate with the endocrine system. *The midbrain contains the inferior and superior colliculi, which receive information from the auditory system and visual sensory input, respectively. The hindbrain primarily controls balance, motor coordination, vital functions, arousal and alertness.

A researcher studies rats in her lab. She presents a green light followed by a treat, and a red light followed by nothing. Which of the following best describes the observation that the rat eventually salivates only when presented with the green light and not the red light? A. Acquisition B. Generalization C. Discrimination D. Negative reinforcement

Correct answer: C To answer this question, we must know associative learning. Remember that there are two definitions of discrimination. The one most often thought of is discrimination as prejudicial attitudes that cause individuals of a particular group to be treated differently from others. Discrimination in classical conditioning is the process by which two similar but distinct conditioned stimuli produce different responses. This is what the question stem describes and therefore is the best answer to the question. There are no negative reinforcers which increase the frequency of a behavior by removing something unpleasant, because there is nothing that occurs with the red light. Generalization in classical conditioning is the process by which two distinct but similar stimuli come to produce the same response. This is the opposite of what occurs in the question. Acquisition refers to the process by which the conditioned stimulus becomes associated with the unconditioned stimulus and begins to evoke the conditioned response. The rat goes through an acquisition phase as the green light begins to evoke salivation; however, this is not the best answer for this question, because it only explains half of the observation and does not explain the rat's response to the red light. (C) is thus the correct answer. step by step Simplify the question The key fact to take from the question stem is that the researcher was applying two similar stimuli, but the rat was showing a different response. The question can be restated as: What is it called when a subject can differentiate between two similar stimuli? What is known? Unknown? We know that the researcher is applying two similar stimuli (lights) and seeing if the rats can learn to differentiate between the colors of lights. We are looking, therefore, for a term that refers to the response of a test subject to more than one stimulus. What is your prediction? There are two terms that refer to how a subject responds to two similar stimuli. Generalization (B) occurs when a second similar stimulus elicits the same response as the first stimulus. Discrimination (C) occurs when two similar stimuli elicit two different responses. (C) is correct.

A fundoscope is a medical instrument that allows physicians to see past the lens and into the back of the eye, which can help screen for retinopathy in diabetic patients. Which of the following features would a physician NOT be able to examine during a fundoscopy? A. Retina B. Optic disk C. Ciliary muscle D. Vitreous humor

Correct answer: C While this question is asking about an instrument used during physical exams, it is really testing basic knowledge about the anatomy of the eye. Both the retina and optic disk are located in the back of the eye and would be able to be visualized by fundoscopy. The vitreous humor is a clear liquid that bathes the inside of the eye and is normally transparent. If there was any cloudiness in the liquid, a physician would not be able to see the back of the eye clearly and would note this abnormality. Lastly, the ciliary muscle is located in the front of the eye around the lens and would not be seen using a fundoscope, making (C) correct.

An owner sprays her dog with a water bottle every time the dog tries to jump up on her couch. The dog eventually learns not to jump on furniture. In terms of operant conditioning, this is an example of: A. positive reinforcement. B. negative reinforcement. C. positive punishment. D. negative punishment..

Correct answer: C Punishment and reinforcement refer to whether the teacher is attempting to increase or decrease the likelihood of a behavior occurring. In this case, the teacher (owner) wants to decrease the likelihood of her dog jumping on the couch and is therefore using punishment (spray bottle). Thus (A) and (B) are incorrect. Positive and negative refer to whether the teacher is adding an insult or taking something away. Since the owner is adding an insult as opposed to taking away something the dog desires, the owner is using positive punishment (C) .

125 of the 600 students in an elementary school experience severe, blood bowel movements at approximately the same time. An investigator believes it to be due to food served in the cafeteria. Which of the following types of studies would be most appropriate to test the hypothesis? A. Randomized control trial B. Longitudinal study C. Cross-sectional study D. Case-control study

Correct answer: D A case-control study start by identifying the number of subjects with or without a particular outcome, and then look backwards to assess how many subjects in each group had exposure to a particular risk factor. Because the question presents a situation in which there are diseased cases and non-diseased controls with an as of yet unknown frequency of exposure to a certain risk factor, the most appropriate type of study to test the investigator's hypothesis is a case-control study. (D) is correct.

Traditional gender theory stated that masculinity and femininity were two extremes on a continuum. Which best explains why this is no longer considered to be true? A. Individuals can have a gender identity that does not match their biological sex B.Individuals can be sexually oriented towards both men and women C. Individuals can be what many cultures consider to be a third gender D. Individuals can be high in both masculinity and femininity

Correct answer: D Androgynous individuals are those high in both masculinity and femininity. The presence of such individuals challenges traditional gender theory, or the idea that gender is a spectrum with two ends. It should be noted that traditional gender theory focuses specifically on gender identity. Gender identity does not necessarily coincide with biological sex or sexual orientation. Therefore, (A) must be incorrect because it refers to discrepancies regarding biological sex, and (B) must be incorrect because it refers to discrepancies regarding sexual orientation. (C) can be eliminating by thinking critically about the implications of a bipolar scale between masculinity and femininity. It is possible, for example, that an individual who identifies as a third gender might be in the middle of the continuum, showing moderate amounts of both masculinity and femininity. The correct answer is (D).

Proponents of the James-Lange theory of emotion and the Cannon-Bard theory of emotion are most likely to agree on which of the following statements? A. The thalamus produces physiological and emotional responses to stimuli almost simultaneously. B. Emotions are experienced as a consequence of a cognitive interpretation of an event. C. Individuals will explain a state of physiological arousal by means of cognition and available environmental stimuli. D. Feelings of emotion are accompanied by physiological responses in the presence of a stimulus.

Correct answer: D Both James-Lange theory and Cannon-Bard theory agree that the perception of events leads to physiological and emotional response. The theories differ on whether physiological arousal from the viscera precedes emotion (James-Lange) or the two happen simultaneously as a result of activity in the thalamus (Cannon-Bard). Both however can support the claim that emotions and physiological responses are mutually occurrent. (D) is, therefore, correct.

If the 95% confidence interval for the proportion of women in a rural community over 60 who have diabetes is 0.40 to 0.60, then which of the following statements is also true? A. 95% of the women in the population have a blood sugar that is 40% to 60% too high B. 95% of the women in the sample have a blood sugar that is 40% to 60% too high C. We can be fairly confident (~95%) that the proportion of women in the sample who have diabetes is between 0.40 and 0.60 D. We can be fairly confident (~95%) that the proportion of women in the population who have diabetes is between 0.40 and 0.60

Correct answer: D Confidence intervals are a range of values determined from the sample mean and standard deviation that are used to estimate the population mean. Rather than finding a p-value, we begin with a desired confidence level (95% is standard) and use a table to find its corresponding z- or t-score. The z- or t-score is then multiplied by the standard deviation, and then added to and subtracted from the mean to create a range of values. Thus, (D) is correct. In depth: For example, to determine the mean age of a population, a sample can be drawn. If the mean age and standard deviation of the sample are determined to be 30 and 3 respectively, and a 95% confidence interval is desired, the next step is to find the corresponding z-score of 1.96. Thus, the range is 30 - (3)(1.96) = 24.12 to 30 + (3)(1.96) = 35.88. Therefore, we can be 95% confident that the true mean age of the population from which this sample is drawn is between 24.12 and 35.88. Again, (D) is correct.

All of the following are consequences of the "aging" U.S. population EXCEPT: A. increased responsibilities of younger generations to care for the older generations. B. increases in the prevalence of chronic illnesses. This answer is incorrect Increased prevalence of chronic illnesses is characteristic of an aging population because older individuals are more likely to experience chronic, versus acute, conditions. C. decreases in the size of the U.S. workforce. D. increased need to find more reliable and healthy food sources.

Correct answer: D Currently, the largest growing cohort of the U.S. population is the 85-or-older cohort. These people generally have higher healthcare needs and are likely retired, which creates a stronger responsibility of the generation after them to provide support. This sense of responsibility is stronger than it was in the past because the older population is continuing to increase relative to other age groups. Therefore, (A) is a true statement and is therefore incorrect. In past decades there has been a shift in the leading causes of death in the elderly from infectious diseases and acute illnesses to chronic diseases and degenerative illnesses. This makes (B) true and therefore incorrect, since a larger elderly population will probably mean a higher prevalence of chronic conditions. (C) is true because more people will be leaving the workforce than entering it. (D) is incorrect, and therefore the correct answer, because the U.S. generally has reliable food sources. The effect of the aging population on food in the country, if there is such an influence, is less pronounced than the other answer choices. (D) is correct.

According to the Demographic Transition Model, in a country experiencing the early stages of industrialization: A. the birth rate, death rate, and population are stabilized. B. the death rate exceeds the birth rate, and the population decreases. C. the birth rate and death rate decrease in parallel, and population remains unchanged. D. the birth rate exceeds the death rate, and the population increases.

Correct answer: D Demographic transition—changes in birth and death rates in a country as it develops from a preindustrial to industrial economic system Demographic transition stage 1—preindustrial society; birth and death rates are both high Demographic transition stage 2—improvements in healthcare, nutrition, sanitation, and wages cause death rates to drop Demographic transition stage 3—improvements in contraception, women's rights, and a shift from an agricultural to an industrial economy cause birth rates to drop; children go to school for many years and may need to be supported by parents for a longer period of time than was formerly the case; families thus have fewer children Demographic transition stage 4—an industrialized society; birth and death rates are both low

A woman notices that her son is acting strangely: he moves as if in a dream and has trouble completing everyday tasks. He confesses that he feels like a robot that has been programmed and gets confused by his own reflection in a mirror. After ruling out the presence of delusions or hallucinations, the most likely explanation for these symptoms is: A. dissociative identity disorder. B. major depressive disorder. C. somatic symptom disorder. D. depersonalization/derealization disorder.

Correct answer: D Dissociative identity disorder—The occurrence of two or more personalities that take control of a person's behavior Major depressive disorder—A mood disorder characterized by at least one major depressive episode Somatic symptom disorder—Involves at least one somatic symptom, which may or may not be linked to an underlying medical condition, that causes disproportionate concern Depersonalization/derealization disorder—Involves feelings of detachment from the mind and body or from the environment

A neuroscientist uses electric shock to condition mice to fear a red light. The neuroscientist then administers a drug that inhibits the sympathetic response, and notes that the mice no longer display a fear response when the light is turned on. Which of the following theories of emotion is best supported by this result? A. Opponent-process theory B. Schachter-Singer theory C. Bottom-up processing D. James-Lange theory

Correct answer: D James-Lange—A stimulus results first in physiological arousal, which leads to a secondary response in which the emotion is labeled Cannon-Bard theory—The cognitive and physiological components of emotion occur simultaneously and result in the behavioral component of emotion, or action Schachter-Singer theory (also called cognitive arousal theory or two-factor theory)—Both arousal and the labeling of arousal based on environment must occur in order for an emotion to be experienced In addition to these theories of emotion, the question references opponent-process theory and bottom up processing. Opponent-process theory—When a drug is taken repeatedly, the body will attempt to counteract the effects of the drug by changing its physiology Bottom up processing (inductive reasoning)—Seeks to create a theory via generalizations by starting with specific instances and then drawing a conclusion from them. The questions also references the sympathetic response. Sympathetic nervous system—The part of the autonomic nervous system activated by stress and is closely associated with rage and fear reactions, also known as "fight or flight" reactions *The theories of emotion are summarized in the table above. The James-Lange theory suggests that a stimulus results first in physiological arousal, which leads to a secondary response in which the emotion is labeled. When the neuroscientist suppresses sympathetic response, the mice no longer exhibit the fear response. The subsequent lack of fear means that the physiological symptoms were required for the emotion to occur originally. Because there is only emotion following physiological response, we can match (D).

Which of the following explains the judgment that a person who walks on the left side of a hallway when everyone else keeps to the right is being rebellious? A. Distinctiveness cues B.Consistency cues C. Environmental cues D. Consensus cues

Correct answer: D One aspect of social cognition is explaining the behavior of others which is mediated by the use of cues. The perceiver in the scenario uses consensus cues to judge the target by comparing the target to everyone else that he sees; therefore, (D) is correct. step by step Simplify the question This question asks us to explain people's behavior as related to the cues that we perceive. In this case, everyone walks on the right side of the hallway with the exception of just one person who walks on the left side of the hallway. This individual is subsequently judged as being rebellious. We can therefore rephrase the question as: Which type of cue is used when someone's behavior differs from other people's behavior? What is known? Unknown? People use a variety of cues to understand and explain the behavior of other people. Within psychology, we distinguish between three different types of cues. As a first step, let's examine these types of cues and their specific characteristics. Consensus cues—Extent to which person's behavior differs from others; if person deviates from socially expected behavior, we are likely to form dispositional (internal) attribution about person's behavior; for example, if person acts different from majority of people we may conclude that person is attention-seeker Consistency cues—Refers to consistent behavior of person over time; the more regular the behavior, the more we associate that behavior with the motives of person; for example, we might assume that fellow student who always shares her classroom notes with other students is nice, helpful, and caring person Distinctiveness cues—Refers to extent to which person engages in similar behavior across series of scenarios; if person's behavior varies in different scenarios, we are more likely to form situational (external) attribution to explain it; for example, someone only has alcoholic drink while socializing with friends during dinner Now that we have familiarized ourselves with the different kinds of cues and their specific characteristics, it becomes clear that environmental cues (C) are not a type of cue that is used to understand and explain behavior. We can therefore eliminate (C). What is your prediction? We are dealing with a type of situation in which someone's behavior is different from the behavior of the majority of other people. The scenario in the question stem matches consensus cues (D). We don't have any information about the behavior of the person over time, hence consistency cues is incorrect and we can eliminate (B). We are presented with a scenario about just one specific situation which means that distinctiveness cues (A) don't apply. To reiterate, only (D) is correct.

A new smartphone application automatically deducts twenty-five cents from the user's bank account every time he or she hits the snooze button on his or her phone's alarm in the morning. The money is automatically deposited in a friend's bank account. With respect to the action of hitting the snooze button, this smartphone application is most likely an example of: A. a positive reinforcement. B. a positive punishment. C. a negative reinforcement. D. a negative punishment.

Correct answer: D Positive Reinforcement—Following a desired behavior, give a desirable consequence Negative Reinforcement—Following a desired behavior, take away an undesirable consequence Positive Punishment—Following an undesirable behavior, give an undesirable consequence Negative Punishment—Following an undesirable behavior, take away a desirable consequence Reinforcement is a consequence meant to encourage the continuation of a certain behavior. Punishment is a consequence meant to discourage a certain behavior. The words "negative" and "positive" in these definitions refer to whether a stimulus (the consequence) is added or taken away as a result of the behavior.

A police officer interviews a witness to a bank robbery, but the witness has difficulty remembering details about the event. After the officer gives the witness details of what the suspect was wearing, the witness begins to remember more about the robbery. Which term best describes this situation? A. Sensitive period B. Reciprocal determinism C. Misinformation effect D. Spreading activation

Correct answer: D Sensitive period—A developmental stage where you are quicker to learn a particular skill (language development, music, reading) Misinformation effect—A phenomenon in which memories are altered by misleading information provided at the point of encoding or recall Spreading activation—The brain stores information in a network by linking concepts with similar characteristics and meaning. Spreading activation is the unconscious activation of closely linked nodes of that network. The answer choices also mention one concept unrelated to memory: Reciprocal determinism—The idea that our environment is molded by our personality and in return, that environment shapes our thoughts, feelings and behaviors

In a random association test, participants are asked to say the first three things that come to their mind when they hear a given word. In one instance, the researcher says "picnic" and the participant responds with the words: "blanket," "ants," and "watermelon." What psychological phenomenon is this researcher most likely testing? A. Assimilation B. Heuristics C. Information processing D. Schemata

Correct answer: D Simplify the question The question provides a research study during which a participant is asked to provide word association in response to an initial word. The question is seeking the psychological phenomenon represented in the study. One way of rewording the question would be: Which of the following is the psychological phenomenon that represents the associations a person has between concepts? What is known? Unknown? Each of the terms listed is known and can be defined. Assimilation—The process of classifying new information into existing schemata Heuristics—Simplified principles used to make decisions; think "rules of thumb" Information processing—A decision-making or problem-solving model in which thinking requires sensation, encoding, and storage of stimuli Schemata—Associations one has among information, concepts, behaviors, etc. There is nothing further that is needed to answer this question. What is your prediction? The research study involves presenting a participant with a given word and asking for three other words that come to mind. The researcher wants to see what associations the participant already holds for the given term. This is consistent with schemata. Predict (D) as correct. Notice in this case that the test the researcher gives is similar to how one might test a semantic network which refers to how the brain physically stores related ideas. A schema (plural: schemata), however, has a broader definition. It is the mental framework we build for ourselves to relate concepts, behaviors, or sequences of events. Unlike semantic networks, a schema doesn't presume that there is a close physical connection in the brain between its nodes. In depth: A schema can include a concept (What is a dog?); a behavior (What do you do when someone asks you your name?); or a sequence of events (What do you normally do in a sit-down restaurant?). The researcher in our question is examining existing associations an individual has with specific concepts. (D) is correct. As a child develops, new information has to be placed into the different schemata. Piaget theorized that new information is processed via adaptation. According to Piaget, adaptation to information comes about by two complementary processes: assimilation and accommodation. Assimilation (A) is the process of classifying new information into existing schemata. If the new information does not fit neatly into existing schemata, then accommodation occurs. Accommodation is the process by which existing schemata are modified to encompass this new information. Heuristics (B) are simplified principles used to make decisions; they are colloquially called "rules of thumb." The availability heuristic is used when we try to decide how likely something is based on how easily similar instances can be imagined. The representativeness heuristic involves categorizing items on the basis of whether they fit the prototypical, stereotypical, or representative image of the category. Information processing (C) has four key components or pillars. First, thinking requires sensation, encoding, and storage of stimuli. Second, stimuli must be analyzed by the brain (rather than responded to automatically) to be useful in decision-making. Third, decisions made in one situation can be extrapolated and adjusted to help solve new problems (also called situational modification). Fourth, problem-solving is dependent not only on the person's cognitive level but also on the context and complexity of the problem. This experiment was not about decision-making or problem-solving, so (C) is incorrect. Only (D) is the correct answer.

An individual hears a persistent loud beeping after turning on his car. Once he puts on his seat belt, the beeping stops. This alarm is an example of: A. positive punishment. B. positive reinforcement. C. negative punishment. D. negative reinforcement.

Correct answer: D Something unpleasant (the beeping) is being removed when the behavior (fastening the seat belt) is performed in order to increase the frequency of the behavior. (D) is correct. Positive Reinforcement—Following a desired behavior, give a desirable consequence Negative Reinforcement—Following a desired behavior, take away an undesirable consequence Positive Punishment—Following an undesirable behavior, give an undesirable consequence Negative Punishment—Following an undesirable behavior, take away a desirable consequence

Which of the following best describes a man who is predominantly heterosexual, but occasionally has homosexual relationships? A. The man's biological disposition best explains his behavior B. The man has a score of 5 on the Kinsey scale C. The man possesses intersex characteristics D. The man has a score of 2 on the Kinsey scale

Correct answer: D The Kinsey scale was a model used to asses sexuality. On a scale from 0 (exclusively attracted to members of the opposite sex) to 6 (entirely attracted to members of the same sex), a person's previous sexual experiences or responses at a current time are used to assign a score. Here, the man is heterosexual but occasionally has homosexual relationships. Therefore, the correct answer cannot be zero, but should be between 1 and 3, with 3 indicating bisexuality. This makes (D) the correct answer. Biological disposition (A) is possibly a contributor to sexual orientation, but is certainly not the only influence and cannot entirely explain sexual orientation. (C) is not necessarily true and cannot be supported given the question stem, nor does it explain the man's sexual orientation. Only (D) is correct.

In a study on the determinants of artistic ability, researchers compared artistic works collected from three groups: identical twins raised together (It), identical twins raised apart (Ia), and fraternal twins raised together (Ft). Four measures of artistic ability were recorded. Which result would lend the most support to the hypothesis that artistic ability is primarily determined by genetics? A. Group It scored more similarly than did group Ia. B. Group Ft scored more similarly than did group Ia. C. Group It scored more similarly than did group Ft. D. Group Ia scored more similarly than did group Ft.

Correct answer: D To better measure genetic effects relative to environmental effects, researchers compare traits in twins raised together to twins raised apart. Regarding characteristics for which there is a dominant genetic component, monozygotic (identical) twins who are raised apart are expected to be more similar than dizygotic (fraternal) twins who are raised together, and approximately equally as similar as monozygotic twins who are raised together. Any answer choice wherein the scores of group Ia are as similar—or more similar—to each other as the scores of the other two groups will therefore support the hypothesis that artistic ability is primarily determined by genetics. (A) and (B) provide supporting evidence for the influence of environment on the development of artistic ability, so they contradict the hypothesis in the question and are therefore incorrect. Both (C) and (D) lend support to the hypothesis in question by demonstrating that twins who are genetically identical are also more similar in terms of artistic ability than are twins who are not genetically identical. However, (D) lends more support to the hypothesis by suggesting that genetics are the primary determinant of artistic ability even when differences in environment come into play. (D) is the best answer.

Which of the following suggests a reasonable relationship between globalization and public health? A. Fluctuations in employment have led to considerable migration. B. Globalization has allowed political interdependence, which has led to increased welfare programs. C. Globalization allows diverse food options in most areas, contributing to better diets in most nations. D. Increased international trade has allowed pathogens to spread to new geographic regions.

Correct answer: D globalization is the process of integrating a global economy with free trade and tapping of foreign labor markets. Global inequality has been further exacerbated by an unprecedented large population spike, placing strain on the world's resources. Public health deals with pathogens that can cause infectious diseases among other things.

A doctor is promoted to chief resident in her hospital. She attributes this to the recent string of good deeds she has done. This example is most consistent with: A. self-enhancement. B. fundamental attribution error. C. the halo effect. D. the just-world hypothesis.

Correct answer: D self-enhancement—the idea that one's successes can be attributed to internal factors and that one's failures can be viewed as based on external factors. It is a strategy aimed at preserving self-worth and self-esteem. As such, it is a part of the self-serving bias fundamental attribution error—the tendency towards making dispositional attributions rather than situational attributions, especially in a negative context. For example, calling someone who isn't able to finish a project prior to the deadline lazy (dispositional), while ignoring the fact that the person may have been sick (situational) halo effect—a cognitive bias in which judgments about a specific aspect of an individual can be affected by one's overall impression of that individual. As such, it is a type of impression bias just-world hypothesis—in a so-called just world, good things happen to good people, and bad things happen to bad people;

Which of the following is a potential reason why children recover more quickly from brain injuries than adults? A. Synaptic pruning B. Long-term potential C. Elaborative rehearsal D. Neuroplasticity.

Correct answer: D Choice (D) . Neuroplasticity refers to the ability of the brain to reorganize and adapt due to changes or injury. This ability decreases as one ages. It is due to this adaptability that children are better able to recover from brain injuries than adults.

Recent immigrants to the United States are often at a disadvantage when seeking a job since they do not know many people in the community. Living in an area with other individuals from the same nationality helps to connect these individuals, thus increasing which of the following? A. Power B. Socioeconomic status C. Ascribed status D. Social capital.

Correct answer: D Choice (D) . Social capital refers to the social networks individuals have in their community, which confers certain advantages. The question details how recent immigrants often have difficulty obtaining employment due to a lack of connections. Thus, if these networks were increased, their social capital would be said to increase.Choice (A) . Power refers to the ability to influence people through real (or perceived) rewards and punishments.Choice (B) . Socioeconomic status is an indicator of wealth or position in a society.Choice (C) . Ascribed status refers to a position in a society based on characteristics beyond the individual's direct control; such as race, gender, ethnicity, and family background.

Which one of the following is NOT one of Hill's criteria for evaluating likelihood of causality in a relationship between two variables? A. Temporality B. Dose-response C. Consistency D. Acceptability.

Correct answer: D Hill's criteria for causality are: temporality, strength, dose-response, consistency, plausibility, consideration of alternate explanations, experiment, specificity, and coherence. Acceptability is not one of these so choice (D) is correct.

Suppose 25 students took a test and the median score was 75%, how many students scored higher than 75%? A. 11 B. 12 C. 13 D. Cannot be determined.

Correct answer: D The median of a data set is the middle number when the data set is arranged in increasing order. With the given median and number of students, it can be concluded that the 13th person scored 75% and that there are 12 students who scored as well or better. However, the number of students that specifically scored better is unknown. Therefore, choice (D) is correct.

Which of the following study types would help control for the placebo effect? A. Ecological study B. Open trial C. Experimental drug therapy D. Double blind study.

Correct answer: D The placebo effect occurs when a patient thinks they are receiving a treatment and improves even though they are actually receiving the control. To help control for this, the patients can't know which drug they are getting and it can be useful for the treatment provider to not know as well, so they don't have any bias towards patient outcomes. When neither the patient nor the care provider know if the patient is in the control group or the experimental group, this is called a double blind study or choice (D) .

A patient with cone dystrophy, a rare genetic disease that leads to the loss of cone cells, would be able to see best at which time of day? A. Afternoon B. Morning C. Dusk D. Night with no ambient light

If a patient lacks cone cells, then they would be unable to see color and would be sensitive to bright light; therefore, they would not be able to see well in the afternoon (A) or in the morning (B). To see, they would rely on their rods which are most functional at reduced illumination such as when it is dusk. Note that rods work best under reduced illumination but do not allow humans to see in the dark, thereby eliminating (D). This makes (C) the correct answer. The retina is made up of approximately 6 million cones and 120 million rods. Cones are used for color vision and to sense fine details; they are most effective in bright light and come in three forms which are named for the wavelengths of light as shown below. In reduced illumination, rods are more functional and only allow sensation of light and dark because they all contain a single pigment called rhodopsin. Rods have low sensitivity to details and are not involved in color vision but permit limited night vision. Step by step Simplify the question Although this question offers a diagnosis of cone dystrophy, the definition of the disease itself is already provided as the loss of cone cells; therefore, rather than recalling the symptoms of the disease, the question is asking about the implications of losing one's cone cells. It could be reworded as the following two questions: What are the implications of losing cone cells? How do different light settings affect the resulting vision? What is known? Unknown? The question not only defines cone dystrophy as the genetic loss of one's cone cells, but it also states that cones are used by the eye for vision. It is still necessary to recall basic information about the rods and cones that allow the eye to see. The retina is located in the back of the eye and utilizes two types of photoreceptors (rods and cones) to convert incoming photons of light to electrical signals. Cones are used for color vision and to sense fine details. They are most effective in bright light. Rods are more functional in reduced illumination and only allow sensation of light and dark. They are not involved in color vision. The unknown in this question is how the time of day affects the vision of a person who has lost cone cells. What is your prediction? Since cone dystrophy results in a loss of cone cells, the person would have to rely on their rods instead. Rods work best in reduced illumination, because they tend to "saturate" and become ineffective in bright light, so predict an answer listing a time of day with reduced light. This supports (C).

Which of the following sleep stages is most likely to immediately precede brief middle-of-the-night awakening? A. Stage 1 B. Stage 2 C. Stage 3 D. REM stage

Simplify the question This question is asking for the stage of sleep that is most likely to proceed brief middle-of-the-night awakening. Said in another way: What sleep stage is closest to being in an "awake state" so that it would not be unreasonable to proceed brief middle-of-the-night awakenings? What is known? Unknown? The stages of sleep as seen on EEG are known. REM sleep—Sometimes called paradoxical sleep: the mind appears close to awake on EEG, but the person is asleep; eye movements and body paralysis occur in this stage Stage 1—Light sleep; dominated by theta waves on EEG Stage 2—Slightly deeper sleep; includes theta waves, sleep spindles, and K complexes Stages 3 and 4—Deep SWS; delta waves predominate on EEG; most sleep disorders occur during Stages 3 and 4 non-rapid eye movement (NREM) sleep; dreaming in SWS focuses on consolidating declarative memories What is your prediction? Knowing that in REM sleep the mind appears to be close to being awake on EEG, predict that this will be the sleep stage that is most likely to immediately precede brief middle-of-the-night awakenings; therefore, predict (D). In depth: A sleep cycle refers to a single complete progression through the sleep stages. The makeup of a sleep cycle changes during the course of the night as shown in the hypnogram above. Early in the night, SWS predominates as the brain falls into deep sleep and then into more wakeful states. Later in the night, REM sleep predominates. Notice that brief periods of awakening are often interspersed within periods of REM sleep (D). This is because during REM sleep, arousal levels reach that of wakefulness, but the muscles are paralyzed. Heart rate, breathing patterns, and even EEG findings (beta and alpha waves) mimic wakefulness.

Which of the following most undermines the idea that education increases social mobility? A. Social status is affected by many factors and therefore cannot be solely attributed to education. B. Studies show that schooling patterns tend to reflect class stratification. C. Only those students from economically advantaged backgrounds succeed in school. D. Studies show that average income is strongly correlated to highest education level attained.

The question stem asks the reader to choose the answer which, if true, would weaken the argument that education enhances social mobility. If education does increase social mobility, then poorer students would have the chance to succeed in school. But if only those from economically advantaged groups succeed (C), this undermines idea that education increases social mobility. (A) is too extreme. Saying that education is not the sole reason for an elevation in social status might be true, but this hardly undermines the idea that education can have a role in increasing social mobility. (B) raises the question of whether education is equally available to every social class, but this is beyond the scope of the question. It is presumed that all groups have access to education; the issue is the effect of that education on social mobility. (D) states that the higher the educational level attained, the more likely that one would obtain a higher income and social status. This would actually strengthen, not undermine, the argument that education increases social mobility.

Which of the following best exemplifies the recognition-primed decision model? A. An art historian uses carbon dating and her knowledge of historical methods to recognize a forgery. B. A firefighter knows that a structure is about to collapse thanks to her years of experience in similar situations. C. A student teacher just studied language-based learning disabilities and is able to identify and assist a student with dyslexia. D. An ornithologist correctly identifies a barred owl without seeing it based solely on his memory of its unique call.

step by step: Simplify the question This question is asking for an example of the cognition-primed decision model. Recall that it's possible whenever there is a MCAT question asking for an example to be given that the example might not be perfect, but it will be pretty close to a prototypical example of what it is exemplifying; therefore, this question might also be reworded to ask: Which of the following answer choices includes a prototypical example of the recognition-primed decision model? What is known? Unknown? It is known that the recognition-primed decision model is a decision-making model in which experience and recognition of similar situations one has already experienced play a large role in decision-making and actions. It is also one of the explanations for the experience of intuition. What is your prediction? Knowing that the recognition-primed decision model is based on making a decision due to already having experienced a similar situation, predict that the correct answer will contain a notion about knowing something due to experiencing a familiar situation. Of the answer choices, this is most consistent with (B), because the firefighter knows that a structure is about to collapse due to her experience with prior collapses. Predict (B). in depth: The recognition-primed decision model (B) is a decision-making model in which experience and recognition of similar situations one has already experienced play a large role in decision-making and actions. It is also one of the explanations for the experience of intuition. Intuition can be defined as the ability to act on perceptions that may not be supported by available evidence. Intuition is often developed by experience. For example, an emergency room physician, over the course of seeing thousands of patients with chest pain, may develop a keen sense of which patients are actually having a heart attack without even looking at an electrocardiogram (EKG) or a patient's vital signs. This intuition can be more accurately described by the recognition-primed decision model: the doctor's brain is actually sorting through a wide variety of information to match a pattern. Over time, the doctor has gained an extensive level of experience that she is able to access without her awareness.


Conjuntos de estudio relacionados

MyProgrammingLab 6.2,6.3,6,4, MPL

View Set

CH 5: The Self-Employment Contributions ACT (SECA)

View Set

Chapter 24 Homework: The Digestive System

View Set

Chapter 1 Fundamentals of Nursing

View Set

Chapter 17 LearnSmart Marketing 021 Ybarra

View Set

PADM 3100 - Workplace Conflict (Quizzes/Test for Entire Course)

View Set